Serology/Immunology (BOC, Harr, Success)

Réussis tes devoirs et examens dès maintenant avec Quizwiz!

79. What is the indicator system used in the complement fixation test? A. Sensitized sheep red blood cells B. Fluorescent-labeled antihuman globulin C. Enzyme-labeled antihuman globulin D. Guinea pig complement

Correct Answer: A The first step in the complement fixation test, the test system, involves the reaction of antibody in the patient's serum to the corresponding antigen in the presence of guinea pig complement. If antibody-antigen binding occurs, complement will bind to the immune complexes. The second step is the addition of sensitized sheep red blood cells (the indicator system). If complement bound to the immune complexes in the first step, it is not available to lyse the sensitized red blood cells. If antibody was not present in the patient sample, complement will not bind to the immune complexes, and it will be free to lyse the sensitized cells.

A patient with ovarian cancer who has been treated with chemotherapy is being monitored for recurrence using serum CA-125, CA-50, and CA15-3. Six months after treatment the CA 15-3 is elevated, but the CA-125 and CA-50 remain low. What is the most likely explanation of these findings? A. Ovarian malignancy has recurred B. CA 15-3 is specific for breast cancer and indicates metastatic breast cancer C. Testing error occurred in the measurement of CA 15-3 caused by poor analytical specificity D. The CA 15-3 elevation is purious and probably benign

Correct Answer: A

A patient's abnormal lymphocytes are positive for CD2 antigen, lack C3 receptors, and are negative for surface immunoglobulin. This can be classified as a disorder of: A. T cells B. B cells C. monocytes D. natural killer cells

Correct Answer: A

A specimen appears to have a perinuclear staining pattern in an antineutrophil cytoplasmic antibody (ANCA) immunofluorescent assay using ethanol fixed neutrophils, suggesting the possibility of a pANCA. On which of the following substrates would this specimen display cytoplasmic speckling? A. Formalin-fixed neutrophils B. Unfixed neutrophils C. HEp-2 cells D. Rabbit kidney tissue

Correct Answer: A

A streptozyme test was performed, but the result was negative, even though the patient showed clinical signs of a streptococcal throat infection. What should be done next? A. Either ASO or anti-deoxyribonuclease B (anti-DNase B) testing B. Another streptozyme test using diluted serum C. Antihyaluronidase testing D. Wait for 3-5 days and repeat the streptozyme test

Correct Answer: A

An 18-month-old boy has recurrent sinopulmonary infections and septicemia. Bruton's X-linked immunodeficiency syndrome is suspected. Which test result would be markedly decreased? A. Serum IgG, IgA, and IgM B. Total T-cell count C. Both B- and T-cell counts D. Lymphocyte proliferation with phytohemagglutinin stimulation

Correct Answer: A

An ANA test on HEp-2 cells shows nucleolar staining in interphase cells and dense chromatin staining in mitotic cels. The most likely cause of this staining patter is: A. Antifibrillarin antibody B. Antiribosomal p antibody C. A serum with nucleolar and homogenours patterns D. Technical artifact

Correct Answer: A

An acute phase protein that binds to the membrane of certain microorganisms and activates the complement system is: A. C-reative protein B. tumore necrosis factor alpha C. neutrophils D. kinins

Correct Answer: A

An immunofluorescence test using reagent antibody directed against the CD3 surface marker would identify which of the following cell types in a sample of human peripheral blood? A. all mature T lymphocytes B. T helper lymphocytes only C. cytotoxic T lymphocytes only D. T regulatory cells only

Correct Answer: A

Anti-extractable nuclear antigens are most likely associated with which of the following anti-nuclear antibody immunofluorescent patterns? A. speckled B. rim C. diffuse D. nucleolar

Correct Answer: A

Antibodies directed at native DNA are most frequently associated with which pattern of fluorescence in the IFA-ANA? A. rim B. diffuse C. speckled D. centromere

Correct Answer: A

Antibodies to thyroid peroxidase can be detected by using agglutination assays. Which of the following diseases may show positive results with this type of assay? A. Graves' disease and Hashimoto's thyroiditis B. Myasthenia gravis C. Granulomatous thyroid disease D. Addison's disease

Correct Answer: A

Antibodies to which of the following immunoglobulins is known to have produced anaphylactic reactions following blood transfusion? A. IgA B. IgD C. IgE D. IgG

Correct Answer: A

Antibody class and antibody subclass are determined by major physiochemical differences and antigenic variation found primarily in the: A. constant region of heavy chain B. constant region of light chain C. variable regions of heavy and light chains D. constant regions of heavy and light chains

Correct Answer: A

Autoantibodies in the absence of Sm are found in patients with: A. mixed connective tissue disease B. systemic lupus erythematosus C. Crohn disease D. multiple myeloma

Correct Answer: A

Biological false-positive VDRL reactions are frequently encountered in patients with: A. lupus erythematosus B. acquired immune deficiency syndrome (AIDS) C. gonorrhea D. tertiary syphilis

Correct Answer: A

Delayed hypersensitvity may be induced by: A. contact sensitivity to inorganic chemicals B. tranfusion reaction C. anaphylactic reaction D. bacterial septicemia

Correct Answer: A

Flocculation tests for syphilis detect the presence of: A. reagin antibody B. antigen C. hemolysin D. Forssman antigen

Correct Answer: A

Given a heterophile antibody titer of 224, which of the following results indicate IM? Absorption with Guinea Pig Kidney; Absorption with Beef Cells A. Two-tube titer reduction; Five-tube titer reduction B. No titer reduction; No titer reduction C. Five-tube titer reduction; Five-tube titer reduction D. Five-tube titer reduction; No titer reduction

Correct Answer: A

In an indirect ELISA method designed to detect antibody to the rubella virus in patient serum, the conjugate used should be: A. anti-human IgG conjugated to an enzyme B. anti-rubella antibody conjugated to an enzyme C. rubella antigen conjugated to an enzyme D. anti-rubella antibody conjugated to a substrate

Correct Answer: A

In assessing the usefulness of a new laboratory test, sensitivity is defined as the percentage of: A. positive specimens correctly identified B. false-positive specimens C. negative specimens correctly identified D. false-negative specimens

Correct Answer: A

In immunofixation electrophoresis: A. the antibody reacts with the antigen and then the complex is electrophoresed B. the antigen is electrophoresed into an antibody containing gel C. the antigen is electrophoresed and the monospecific antisera is reacted with it D. the antigen is electrophoresed, transferred to nitrocellulose and then antibody reacts with it and an EIA is preformed

Correct Answer: A

In primary biliary cirrhosis, which of the following antibodies is seen in high titers? A. antimitochondrial B. anti-smooth muscle C. anti-DNA D. anti-parietal cell

Correct Answer: A

In skin test, a wheal and flare development is indicative of: A. immediate hypersensitivity B. delayed hypersensitivity C. anergy D. Arthus reaction

Correct Answer: A

In the cold agglutinin test, the tubes contain the serum and erythrocytes are allowed to stand overnight in the refrigerator, and the results are read the next morning. If a disk of the erythrocytes float up from the bottom of the tube with only the flick of finger, this is read as a: A. 4+ reaction B. 2+ reaction C. 1+ reaction D. negative reaction

Correct Answer: A

In the direct fluorescent antibody test for primary syphilis, spirochetes are detected by addition of labeled antibody to? A. Treponema pallidum B. cardiolipin C. human immunoglobulin D. nonpathogenic treponemes

Correct Answer: A

Infantile X-linked agammaglobulinemia is referred to as: A. Bruton agammaglobulinemia B. DiGeorge syndrome C. Swiss-ype agammaglobulinemia D. ataxia telangiectasia

Correct Answer: A

Interpret the following ASO results: -Tube Nos. 1-4 (Todd unit 125): no hemolysis -Tube No. 5 (Todd unit 166): hemolysis A. Positive Todd unit 125 B. Positive Todd unit 166 C. No antisteptolysin O present D. Impossible to interpret

Correct Answer: A

Interpret the following quantitative RPR test results: -RPR: weakly reactive (1:8); reactive (1:8-1:64) A. Excess antibody, prozone effect B. Excess antigen, postzone effect C. Equivalence of antigen and antibody D. Impossible to interpret; testing error

Correct Answer: A

Macrophage phagocytosis of bacteria is enhanced by which of the following: A. opsonin B. antigen C. hapten D. secretory piece

Correct Answer: A

Macrophages are characterized by: A. surface receptors for C3b B. surface CD3 expression C. in vitro synthesis of immunoglobin D. large amounts of rough endoplasmic reticulum

Correct Answer: A

Positive rheumatoid factor is generally associated with: A. hyperglobulinemia B. anemia C. decreased erythrocyte sedimentation rate D. azotemia

Correct Answer: A

Refer to the following flow cytometric data: absolute WBC: 8,930 total lymphocytes: 30% B lymphocytes: 40% T lymphocytes: 58% Calculate the absolute count for B lymphocytes. A. 1,072 B. 2,679 C. 3,572 D. 6,251

Correct Answer: A

Refer to the following illustration of the hepatitis B virus: See BOC pg 246 Pic 1 Select the corresponding lettered component indicated on the diagram for surface antigen. A. A B. B C. C D. D

Correct Answer: A

Refer to the following illustration: See BOC pg 238 Pic 2 Which of the above figures demonstrates a reaction pattern of identity? A. Figure #1 B. Figure #2 C. Figure #3 D. Figure #4

Correct Answer: A

Rheumatoid factor in a patient's serum may cause a false: A. positive test for the detection of IgM class antibodies B. negative test for the detection of IgM class antibodies C. positive test for the detection of IgG class antibodies D. negative test for the detection of IgG class antibodies

Correct Answer: A

Rhuematoid factors are immunoglobulins with specificity for allotypic determinants located on the: A. Fc fragment of IgG B. Fab fragment of IgG C. J chain of IgM D. secretory of component of IgA

Correct Answer: A

Select the best donor for a man, blood type AB, in need of a kidney transplant. A. His brother, type AB, HLA matched for class II antigens B. His mother, type B, HLA matched for class I antigens C. His cousin, type O, HLA matched for class II antigens D. Cadaver donor, type O, HLA matched for some class I and II antigens

Correct Answer: A

Systemic lupus erythematosus patients often have which of the following test results? A. high titers of DNA antibody B. decreased serum immunoglobulin levels C. high titers of anti-smooth muscle antibodies D. high titers of antimitochondrial antibody

Correct Answer: A

The HLA antibodies used in histocompatibility typing have been obtained form which of the following? A. multiparous women B. nonidentical siblings C. sheep blood D. rabbit serum

Correct Answer: A

The J-chain is associated with which of the following immunoglobulins? A. IgA B. IgG C. IgE D. IgD

Correct Answer: A

The Rapid Plasma Reagin test: A. is useful in screening for syphilis B. is useful in diagnosing syphilis C. does not give false-positives D. uses heated plasma

Correct Answer: A

The classic antibody response pattern following infection with hepatitis A is: A. increased in IgM antibody; decrease in IgM antibody; increase IgG antibody B. detectable presence of IgG antibody only C. detectable presence of IgM antibody only D. decrease in IgM antibody; increase in IgG antibody of the IgG3 subtype

Correct Answer: A

The curve below was obtained by adding increasing amounts f a soluble antigen to fixed volume of monospecific antiserum: See BOC pg 237 Pic 1 The area on the curve for prozone is: A. A B. B C. C D. D

Correct Answer: A

The directions for a slide agglutination test instruct that after mixing the patient's serum and latex particles, the slide must be rotated for 2 minute. What would happen if the slide were rotated for 10 minutes? A. Possible false-positive result B. Possible false-negative result C. No effect D. Depends on the amount of antibody present in the sample

Correct Answer: A

The following cold aglutinin titer results are observed: Tube #: 1;2;3;4;5;6;7;8;9;10; dilution: 1:1; 1:2; 1:4; 1:8; 1:16; 1:32; 1:64; 1:128; 1:256; 1:512 4C: +; +; +; +; +;+; +; +; 0, 0 37C: 0, 0, 0, 0, 0, 0, 0, 0, 0, 0 The best interpretation is: A. positive, 1:128 B. negative C. invalid because 37 C reading is negative D. repeat the 4C readings

Correct Answer: A

The following results are form a rubella titer performed on acute and convalescent sera using a 2-fold serial dilution: Date tested: 1/24/04 Acute serum titer: 1:8 Convalescent serum titer: 1:32 After evaluating the above results, the best interpretation is: A. results are consistent with active infection with rubella B. variation in the acute serum titers invalidates these results C. test should be repeated by a different technologist D. patient was not infected with rubella

Correct Answer: A

The most commonly used serological indicator of recent streptococcal infection is the antibody to: A. streptolysin B. hyaluronidase C. NADase D. DNA

Correct Answer: A

The normal controls for a quantitative B lymphocyte assay should have a value of what percentage of total lymphocytes counted? A. 21% B. 48% C. 76% D. 89%

Correct Answer: A

The presence of HbsAg, anti-HBc and often HbeAg is characteristic of: A. early acute phase HBV hepatitis B. early convalescent phase HBV hepatitis C. recovery phase of acute HBV hepatitis D. past HBV infection

Correct Answer: A

The profile that matches the typical test profile for chronic acitve hepatits due to hepatitis B virus is: Profile: HBsAg; IgM anti-HBc; anti-HBc; anti-HBs A: +,=,+,= B: +,+,=,= C:=,=,+,= D:=,=,=,+ A. profile A B. profile B C. profile C D. profile D

Correct Answer: A

The result of an anti-nuclear antibody test was a titer of 1:320 with a peripheral pattern. Which of the following sets of results best correlate with these results? A. anti-dsDNA titer 1:80, and a high titer of antibodies to Sm B. antimitochondrial antibody titer 1:160, and antibodies to RNP C. anti-Scl-70, and antibodies to single-stranded DNA D. high titers of anti-SS and anti-B

Correct Answer: A

The serum hemolytic complement level (CH50): A. is a measure of total complement activity B. provides the same information as a serum factor B level C. is detectable when any component of the classical system is congenitally absent D. can be calculated from the serum concentrations of the individual components

Correct Answer: A

Tumor markers found in the circulation are most frequently measure by: A. immunoassays B. thin-layer chromatography C. high-pressure liquid chromatography D. colorimetry

Correct Answer: A

What comprises the indicator system in an indirect ELISA for detecting antibody? A. Enzyme-conjugated antibody + chromogenic substrate B. Enzyme conjugated antigen + chromogenic substrate C. Enzyme + antigen D. Substrate + antigen

Correct Answer: A

What criteria constitute the classification system for HIV infection? A. CD4-positive T-cell count and clinical symptoms B. Clinical symptoms, condition, duration, and number of positive bands on Western blot C. Presence of absence of lymphadenopathy D. Positive bands on Western blot and CD8-positive T-cell count

Correct Answer: A

What has happened in a titer, if tube Nos. 5-7 show a stronger reaction than tube Nos. 1-4? A. Prozone reaction B. Postzone reaction C. Equivalence reaction D. Poor technique

Correct Answer: A

What is an advantage of performing a prostate-specific antigen (PSA) test for prostate cancer? A. PSA is stable in serum and not affected by a digital-rectal examination B. PSA is increased only in prostatic malignancy C. A normal serum level rules out malignant prostatic disease D. The percentage of free PSA is elevated in persons with malignant disease

Correct Answer: A

What kind of antigen-antibody reaction would be expected if soluble antigen is added to homologous antibody? A. precipitation B. agglutination C. complement fixation D. hemagglutination

Correct Answer: A

What screening test should be performed first in a young patient suspected of having an immune dysfunction disorder? A. Complete blood count (CBC) and white cell differential B. Chemotaxis assay C. Complement level D. Bone marrow biopsy

Correct Answer: A

What type of antibodies is represented by the solid or homogenous pattern in the immunofluorescence test for antinuclear antibodies? A. Antihistone antibodies B. Anticentromere antibodies C. Anti-ENA (anti-SM and anti-RNP) antibodies D. Anti-RNA antibodies

Correct Answer: A

What type of disorders would show a decrease in C3, C4, and CH50? A. Autoimmune disorders such as SLE and RA B. Immunodeficiency disorders such as common variable immunodeficiency C. Tumors D. Bacterial, viral, fungal, or parasitic infections

Correct Answer: A

Which IgG subclass is most efficient at crossing the placenta? A. IgG1 B. IgG2 C. IgG3 D. IgG4

Correct Answer: A

Which class of immunoglobulin is though to function as an antigenic receptor site on the surface of immature B lymphocytes? A. IgD B. IgE C. IgA D. IgG

Correct Answer: A

Which disease is least likely when a nucleolar pattern occurs in an immunofluorescence test for antinuclear antibodies? A. MCTD B. Sjogern's syndrome C. SLE D. Scleroderma

Correct Answer: A

Which hepatitis B marker is the best indicator of early acute infection? A. HBsAg B. HBeAg C. Anti-HBc D. Anti-HBs

Correct Answer: A

Which hepatitis B markers should be performed on blood products? A. HBsAg and anti-HBc B. Anti-HBs and anti-HBc C. HBeAg and HBcAg D. Anti-HBs and HBeAg

Correct Answer: A

Which immunologic mechanism is usually involved in bronchial asthma? A. immediate hypersensitivity B. antibody mediated cytotoxicity C. immune complex D.delayed hypersensitivity

Correct Answer: A

Which laboratory technique is most frequently used to diagnose and follow the course of therapy of a patient with secondary syphilis? A. flocculation B. precipitation C. complement fixation D. indirect immunofluorescence

Correct Answer: A

Which of the following activities is associated with C3b? A. opsonization B. anaphylaxis C. vasoconstriction D. chemotaxis

Correct Answer: A

Which of the following conditions will most likely result in a false-negative DAT test? A. Insufficient washing of RBCs B. Use of heavy chain-specific polyclonal anti-human Ig C. Use of excessive centrifugal force D. Use of a sample obtained by finger puncture

Correct Answer: A

Which of the following describes an antigen-antibody precipitation reaction of non-identity? A. precipitin lines cross, forming double spurs B. precipitin lines fuse, forming a single spur C. no precipitin lines are formed D. precipitin lines fuse, forming a single arc

Correct Answer: A

Which of the following describes an antigen-antibody reaction? A. the reaction is reversible B. the reaction is the same as a chemical reaction C. a lattice is formed at prozone D. a lattice is formed at postzone

Correct Answer: A

Which of the following immunoglobulin classes is associated with a secretory component (transport piece)? A. IgA B. IgD C. IgE D. IgG

Correct Answer: A

Which of the following is a description of type 1 hypersensitivity reaction? A. Ragweed antigen cross links with IgE on the surface of mast cells, causing release of preformed mediators and resulting in symptoms of an allergic reaction B. Anti-Fya from a pregnant woman crosses the placenta and attaches to the Fya antigen-positive red cells of the fetus, destroying the red cells C. Immune complex deposition occurs on the glomerular basement membrane of the kidney, leading to renal failure. D. Exposure to poison ivy causes sensitized T cells to release lymphokines that cause a localized inflammatory reaction

Correct Answer: A

Which of the following is an important marker for the presence of immature B cells in patients with acute lymphocytic leukemia (ALL)? A. terminal deoxynucleotidyl transferase (TdT) B. adenosine deaminase C. glucose-6-phosphate dehydrogenase D. purine nucleoside phosphorylase

Correct Answer: A

Which of the following is the "recognition unit" in the classical complement pathway? A. C1q B. C3a C. C4 D. C5

Correct Answer: A

Which of the following is the most sensitive and approriate method for the detection of rheumatoid factor? A. nephelometry B. immunofixation electrophoresis C. immunofluorescence D. manual latex agglutination

Correct Answer: A

Which of the following methods used for HIV identification is considered a signal amplification technique? A. Branched chain DNA analysis B. DNA PCR C. Reverse transcriptase PCR D. Nucleic acid sequence based assay (NASBA)

Correct Answer: A

Which of the following substances, sometimes used as a tumor marker, is increased two- or threefold in a normal pregnancy? A. Alkaline phosphatase (ALP) B. Calcitonin C. Adrenocortocotropic hormone (ACTH) D. Neuron-specific enolase

Correct Answer: A

Which statement best describes immunophenotyping? A. Lineage determination by detecting antigen on the surface of the gated cells using fluorescent antibodies B. Identification of cell maturity using antibodies to detect antigens within the nucleus C. Identification and sorting of cells by front and side-scatter of light from a laser D. Analysis of cells collected by flow cytometry using traditional agglutination reactions

Correct Answer: A

Which statement concerning non-Forssman heterophile antibody is true? A. It is not absorbed by guinea pig antigen B. It is absorbed by guinea pig antigen C. It does not agglutinate horse RBCs D. It does not agglutinate sheep RBCs

Correct Answer: A

Which test are considered screening test for HIV? A. ELISA, 4th generation, and rapid antibody tests B. Immunofluorescence, Western blot, radioimmuno-precipitation assay C. Culture, antigen capture assay, polymerase chain reaction D. Reverse transcriptas and messenger RNA (mRNA) assay

Correct Answer: A

Which test is most likely to be positive in the tertiary stage of syphilis? A. FTA-ABS B. RPR C. VDRL D. Reagin screen test (RST)

Correct Answer: A

Which test is used to evaluate the cellular immune system in a patient? A. skin test for commonly encountered antigens B. determination of isohemagglutinin titer C. immunoelectrophoresis of serum D. measurement of anti-HBsAg after immunization

Correct Answer: A

Which test should be performed when a patient has a reaction to transfused plasma products? A. Immunoglobulin levels B. T-cell count C. Hemoglobin levels D. Red cell enzymes

Correct Answer: A

Which test would best distinguish between SLE and MCTD? A. Multiplex of ELISA test for anti-SM and anti-RNP B. Immunofluorescence testing using Crithidia as substrate C. Slide agglutination testing D. Laboratory tests cannot distinguish between these disorders

Correct Answer: A

Which type of nephelometry is used to measure immune complex formation almost immediately after reagent has been added? A. Rate B. Endpoint C. Continuous D. One dimensional

Correct Answer: A

Why is laboratory diagnosis difficult in cases of Lyme disease? A. Clinical response may not be apparent upon initial infection; IgM antibody may not be detected until 3-6 weeks after the infection B. Laboratory tests may be designed to detect whole Borrelia burgdorferi, not flagellar antigen found early in infection C. Most laboratory test are technically demanding and lack specificity D. Antibodies formed initially to B. burgdorferi may cross react in antigen tests for autoimmune disease

Correct Answer: A

39. The type of immunity that follows the injection of an immunogen is termed A. Artificial adaptive B. Natural adaptive C. Artificial passive D. Natural passive

Correct Answer: A Active immunity follows exposure to an antigen that stimulates the recipient to develop his or her own immune response. Vaccines are an example of artificial immunity in that the animal was exposed to the immunogen by the actions of a healthcare provider (unnatural). Surviving infections can result in natural active immunity. Protection is due to the formation of memory cells.

21. An example of immune injury due to the deposition of antigen-antibody complexes is A. Acute glomerulonephritis B. Bee-sting C. Contact dermatitis D. Penicillin allergy

Correct Answer: A Acute glomerulonephritis is caused by the presence of a soluble circulating antigen (Ag) that provokes and combines with antibody (Ab). As these Ag-Ab complexes reach a critical size, they are deposited in the glomerular membranes of the kidney. Upon deposition, an acute inflammatory reaction occurs because of complement activation. Bee-sting and penicillin allergies are examples of IgE-mediated anaphylactic reactions. Contact dermatitis is mediated by T cells, not antibody.

54. Loss of self-tolerance results in A. Autoimmune disease B. Graft-versus-host disease C. Immunodeficiency D. Tumors

Correct Answer: A The immune system recognizes host cells as self and is tolerant to antigens on those cells. The loss of tolerance will result in an autoimmune disease in which the immune system mounts an immune response against self cells. Graft-versus-host disease occurs when a bone marrow graft is incompatible with the host tissue and attacks the host.

80. The isotype of an immunoglobulin antibody: A. Is defined by the heavy chain B. Is defined as different as alleles of the same antibody type (e.g., IgG) C. Is constant for all immunoglobulins of an individual D. Is the variation within the variable region

Correct Answer: A The isotype of an antibody is determined by which heavy chain is present. The term "idiotype" refers to the variable region of an immunoglobulin molecule. The variable region is the portion of immunoglobulin that binds antigen. Every immunoglobulin with a given antigenic specificity has a unique idiotype.

9. Which of the following is more likely to be diagnostic of an acute infection? A. A total acute antibody titer of 2 followed by a convalescent titer of 16 B. A total acute antibody titer of 80 followed by a convalescent titer of 40 C. A total antibody titer of 80 D. An IgG antibody titer of 80

Correct Answer: A The most significant indicator of acute or recent infection is the presence of a rising antibody liter. A fourfold or greater rise in titer, from 2 to 16, is significant. Even relatively high antibody liters of IgG may indicate past infection. IgM is produced first following infections, so a high IgM titer is also suggestive of an acute infection.

61. The activity of natural killer (NK) cells A. Does not require previous exposure to an antigen B. Involves phagocytosis and killing of bacteria C. Requires interaction with cytotoxic T cells D. Requires interaction with B cells

Correct Answer: A The natural killer (NK) cells destroy target cells through an extracellular nonphagocytic mechanism. NK cells are part of the host's innate resistance and, therefore, do not need previous exposure to an antigen to be active. They also do not need interaction with B or cytotoxic T cells.

100. A 38-year-old woman visited her physician because of fatigue, fever, and joint pain (proximal interphalangeal, wrist, and knee joints). She also noticed sensitivity to the sun and reported having a rash following recent exposure. The physician noted a rash over her nose and cheeks. Laboratory results included white blood cell count 5.5 x 10^9/L (reference range 4.8-10.8 x 10^9/L) and red blood cell count 4.5 x 10^12/L (reference range 4.0-5.4 x 10^12/L). Urinalysis results were within reference ranges, except for 4+ protein and 1+ RBCs, 0-3 hyaline casts/lpf and 0-1 RBC cast/lpf on microscopic examination. Which of the following test would be most helpful in diagnosing this patient's condition? A. Anti-nuclear antibody B. a-Fetoprotein C. Anti-streptolysin O D. Hepatitis profile

Correct Answer: A The presence of arthritis is suggestive of a number of autoimmune diseases. Protein, RBCs, and casts in the urine are indicative of kidney inflammation. These signs and symptoms along with the rash on the face are characteristic of systemic lupus erythematosus (SLE). A commonly used sensitive screening test for SLE is the anti-nuclear antibody (ANA) test. The ANA, however, is not specific for SLE. If the ANA were positive, additional autoantibody tests specific for SLE (e.g., anti-Smith) should be performed.

91. B lymphocytes and T lymphocytes are derived from: A. Hematopoietic stem cells B. Macrophages or monocytes C. Mucosa-associated lymphoid tissue D. The fetal liver

Correct Answer: A The stem cells of the bone marrow give rise to both T and B cells, as well as other cells in the bloodstream. Macrophages and monocytes also arise from hematopoietic stem cells, but they do not differentiate into lymphocytes. Mucosa-associated lymphoid tissue contains mature lymphocytes, particularly B cells, but is not the source of lymphocytes. The fetal liver is a maturation site for B lymphocytes during fetal life but is not the source of those lymphocytes.

32. A molecule found in human serum sometimes used as a tumor marker is A. a-Fetoprotein B. HBsAg C. Biotin D. CD1

Correct Answer: A α-Fetoprotein (AFP) and carcinoembryonic antigen (CEA) are oncofetal antigens that become expressed after malignant transformation. Approximately 70% of patients with primary hepatoma have elevated levels of AFP. However, the major use of determining AFP levels is in monitoring patients undergoing cancer treatment.

1. The interaction between an individual antigen and antibody molecule depends upon several types of bonds such as ionic bonds, hydrogen bonds, hydrophobic bonds, and van der Waals forces. How is the strength of this attraction characterized? A. Avidity B. Affinity C. Reactivity D. Valency 3.2 - Immunologic Procedures

Correct Answer: B

14. What is the purpose of C3a, C4a, and C5a, the split products of the complement cascade? A. To bind with specific membrane receptors of lymphocytes and cause release of cytotoxic substances B. To cause increased vascular permeability, contraction of smooth muscle, and release of histamine form basophils C. To bind with membrane receptors of macrophages to facilitate phagocytosis and the removal of debris and foreign substances D. To regulate and degrade membrane cofactor protein after activation by C3 convertase 3.1 - Basic Principles of Immunology

Correct Answer: B

16. Which region determines whether an immunoglobulin molecule can fix complement? A.Vh B. Ch C. Vl D. Cl 3.1 - Basic Principles of Immunology

Correct Answer: B

18. Which immunoglobulin appears first in the primary immune response? A. IgG B. IgM C. IgA D. IgE 3.1 - Basic Principles of Immunology

Correct Answer: B

2. Which two organs are considered the primary lymphoid organs in which immunocompetent cells originate and mature? A. Thyroid and Peyer's patches B. Thymus and bone marrow C. Spleen and mucosal-associated lymphoid tissue (MALT) D. Lymph nodes and thoracic duct 3.1 - Basic Principles of Immunology

Correct Answer: B

24. Which MHC class of molecule is necessary for antigen recognition by CD4-positive T cells? A. Class I B. Class II C. Class III D. No MHC molecule is necessary for antigen recognition 3.1 - Basic Principles of Immunology

Correct Answer: B

27. The T-cell antigen receptor is similar to immunoglobulin molecules in that it: A. Remains bound to the cell surface and is never secreted B. Contains V and C regions each of its chains C. Binds complement D. Can cross the placenta and provide protection to a fetus 3.1 - Basic Principles of Immunology

Correct Answer: B

28. Toll-like receptors are found on which cells? A. T cells B. Dendritic cells C. B cells D. Large granular lymphocytes 3.1 - Basic Principles of Immunology

Correct Answer: B

4. T cells travel from the bone marrow to the thymus for maturation. What is the correct order of the maturation sequence for T cells in the thymus? A.Bone marrow to the cortex; after thymic education, released back to peripheral circulation B. Maturation and selection occur in the cortex; migration to the medulla; release of mature T cells to secondary lymphoid organs C. Storage in either the cortex or medulla; release of T cells into the peripheral circulation D. Activation and section occur in the medulla; mature R cells are stored in the cortex until activated by antigen 3.1 - Basic Principles of Immunology

Correct Answer: B

5. Which cluster of differentiation (CD) marker appears during the first stage of T-cell development and remains present as an identifying marker for T cells? A. CD1 B. CD2 C. CD3 D. CD4 or CD8 3.1 - Basic Principles of Immunology

Correct Answer: B

8. How are cytotoxic T cells (Tc cells) and natural killer (NK) cells similar? A. Require antibody to be present B. Effective against virally infected cells C. Recognize antigen in association with HLA class II markers D. Do not bind to infected cells 3.1 - Basic Principles of Immunology

Correct Answer: B

A 19-year-old girl came to her physician complaining of a sore throat and fatigue. Upon physical examination, lymphadenopathy was noted. Reactive lymphocytes were noted on the differential, but a rapid test for IM antibodies was negative. Liver enzymes were only slightly elevated. What test(s) should be ordered next? A. Hepatitis testing B. EBV serological panel C. HIV confirmatory testing D. Bone marrow biopsy

Correct Answer: B

A 26-year-old nurse developed fatigue, a low-grade fever, polyarthritis and urticaria. Two months earlier she had cared for a patient with hepatitis. Which of the following findings are likely to be observed in this nurse? A. a negative hepatitis B surface antigen test B. elevated AST and ALT levels C. a positive rheumatoid factor D. a postive Monospot test

Correct Answer: B

A biological false-positive reaction iS least likely with which test for syphilis? A. VDRL B. Fluorescent T. pallidum antibody absorption test (FTA-ABS) C. RPR D. All are equally likely to detect a false-positive result

Correct Answer: B

A child suspected of having an inherited humoral immunodeficiency disease is given diphtheria/ tetanus vaccine. Two weeks after the immunization, his level of antibody to the specific antigens is measured. Which result is expected for this patient if he/she indeed has humoral deficiency? A. Increased levels of specific antibody B. No change in the level of specific antibody C. An increase in IgG-specific antibody but not IgM-specific antibody D. Increased levels of nonspecific antibody

Correct Answer: B

A flow cytometry scattergram of a bone marrow sample shows a dense population of cells located in-between normal lymphoid and normal myeloid cells. What is the most likely explanation? A. The sample was improperly collected B. An abnormal cell population is present C. The laser optics are out of alignment D. The cells are most likely not leukocytes

Correct Answer: B

A patient had surgery colorectal cancer, after which he received chemotherapy for 6 months. The test for carcinoembryonic antigen (CEA) was normal at this time. One year later, the bimonthly CEA was elevated (above 10 ng/mL). AN examination and biopsy revealed the recurrence of a small tumor. What was the value of the results provided by the CEA test in this clinical situation? A. Diagnostic information B. Information for further treatment C. Information on the immunologic response of the patient D. NO useful clinical information in this case

Correct Answer: B

A patient presents with clinical symptoms of celiac disease. Test for anti-tissue transglutaminase and antigliadin antibodies are negative. Which of the following test should be ordered? A. IgG level B. HLA DQ typing C. HLA DR typing D. IgM level

Correct Answer: B

A patient suspected of having toxoplasmosis has a specific IgG antibody titer of 1:64 upon initial testing. The titer of a second serum sample from the patient, take 3 weeks later, was 1:256. These results are indicative of: A. past infection B. present infection C. allergic response D. recent vaccination

Correct Answer: B

A patient with a B-cell deficiency will most likely exhibit: A. decrease phagocytosis B. increased bacterial infections C. decrease complement levels D. increased complement levels

Correct Answer: B

A patient with a T-cell deficiency will most likely exhibit: A. increased immune complex formation B. increased parasitic infections C. decreased IgE-mediated responses D. decrease complement levels

Correct Answer: B

A patient's serum is being analyzed in a sandwich assay. This patient has received mouse monoclonal antibody therapy, and shows a false positive reaction in the sandwich assay, which is due to: A. the mouse antibody in the patient's serum reacting to the antigen B. the presence of human anti-mouse antibody activity C. antibody to a mouse virus D. production of a monoclonal gammopathy of unknown significance after the antibody

Correct Answer: B

A peripheral blood total leukocyte count is 10.0 x 10^3/uL. The differntial reveals 55% neutrophils, 2% eosinophils, 40% lymphocytes and 3% monocytes. Assuming a lymphocyte recovery of 85%-95%, what is the expected number of T cells in a normal individual? A. 750/uL B. 2,500/uL C. 4,000/uL D. 8,000/uL

Correct Answer: B

A pregnant woman come to her physician with a maculopapular rash on her face and neck. Her temperature was 37.7C (100 F). Rubella test for both IgG and IgM antibody were positive. What positive test(s) would reveal a diagnosis of congenital rubella syndrome in her baby after birth? A. Positive rubella tests for both IgG and IgM antibody B. Positive rubella test for IgM C. Positive rubella test for IgG D. No positive test is revealed in congenital rubella syndrome

Correct Answer: B

A request is received in the laboratory for assistance in selecting the appropriate test(s) fro detecting Lyme disease. Which of the following would be suggested? A. Stool culture should be done to isolate the causative organism B. The organism is difficult to isolate, and antibody titers will provide the most help. C. Borrelia burgdoferi is easily isolated from routing blood cultures D. This is an immunologic syndrome, and cultures are not indicated

Correct Answer: B

A serological test for syphilis that depends upon the detection of cardiolipin-lecithin-cholesterol antigen is: A. FTA-ABS B. RPR C. MHA-TP D. TPI

Correct Answer: B

An IFE performed on a serum sample showed a narrow dark band in the lanes containing anti-y and anti-(greek h). How should this result be interpreted? A. Abnormal decreased IgG concentration B. Abnormal test result demonstrating monoclonal IgG(greak h) C. Normal test result D. Impossible to determine without densitometric

Correct Answer: B

An immunofluorescence microscopy assay (IFA) was performed, and a significant antibody titer was reported. Positive and negative controls performed as expected. However, the clinical evaluation of the patient was not consistent with a positive finding. What is the most likely explanation of this situation? A. The clinical condition of the patient changed since the sample was tested B. The pattern of fluorescence was misinterpreted C. The control results were misinterpreted D. The wrong cell line was used for the test

Correct Answer: B

Anti-phospholipid antibodies associated with autoimmune disorders tend to have immunoglobulin (IgG) that belongs to which of the following subclasses? A. IgG1 and IgG3 B. IgG2 and IgG4 C. IgG1 and IgG4 D. IgG2 and IgG3

Correct Answer: B

Antinuclear antibody tests are performed to help diagnose: A. acute leukemia B. lupus erythematosus C. hemolytic anemia D. Crohn disease

Correct Answer: B

Bence Jones proteins are: A. immunoglobulin catabolic fragments in the urine B. monoclonal light chains C. whole immunoglobulins D. Fab fragments of a monoclonal protein

Correct Answer: B

Bone marrow transplant donors and their recipients must be matched for which antigen system? A. ABO-Rh B. HLA C. CD4/CD8 D. P1^(a1)

Correct Answer: B

Cells known to be actively phagocytic include: A. neutrophils, monocytes, basophils B. neutrophils, eosinophils, monocytes C. monocytes, lymphocytes, neutrophils D. lymphocytes, eosinophils, monocytes

Correct Answer: B

Cells that are precursors of plasma cells and also produce immunoglobulins are: A. macrophages B. B lymphocytes C. T lymphocytes D. monocytes

Correct Answer: B

Chronic carriers of HBV: A. have chronic symptoms of hepatitis B. continue to carry HBV C. do not transmit infection D. carry HBV but are not infectious

Correct Answer: B

Components of the complement system most likely to coat a cell are A. C1 and C2 B. C3 and C4 C. C6 and C7 D. C8 and C9

Correct Answer: B

Given a heterophile antibody titer of 224, which of the following results indicate an error in testing? Absorption with Guinea Pig Kidney; Absorption with Beef Cells A. Two-tube titer reduction, Five-tube titer reduction B. No titer reduction; No titer reduction C. Five-tube titer reduction; Five-tube titer reduction D. Five-tube titer reduction; No titer reduction

Correct Answer: B

Given the following data: WBC: 5.0 x 10^3/uL lymphs: 15% CD4: 8% Calculate the absolute CD4: A. 40 B. 60 C. 400 D. 750

Correct Answer: B

HLA typing of a family yields the following results: Locus A; Locus B father: (8, 12); (17, 22) mother: (7, 12); (13, 27) On the basis of these genotypes, predict the possibility of ankylosing spondylitis in this percentage of their children. A. 25% of their children B. 50% of their children C. 75% of their children D. 100%

Correct Answer: B

Hereditary angioedema is characterized by: A. decreased activity of C3 B. decreased activity of C1 esterase inhibitor C. increased activity of C1 esterase inhibitor D. increased activity of C2

Correct Answer: B

How can interfering cold agglutinins be removed form a test sample? A. Centrifuge the serum and remove the top layer B. Incubate the clot at 1C-4C for several hours, then remove serum C. Incubate the serum at 56C in a water bath for 30 minutes D. Use an anticoagulated sample

Correct Answer: B

How is HLA typing used in the investigatioin of genetic diseases? A. For prediction of the severity of the disease B. For genetic linkage studies C. For direct diagnosis of disease D. Is not useful in this situation

Correct Answer: B

Immunoassays are based on the principle of: A. separation no bound and free analyte B. antibody recognition of homologous antigen C. protein binding to isotopes D. production of antibodies against drugs

Correct Answer: B

Immunoglobulin idiotypic diversity is best explained by the theory of: A. somatic mutation B. germ line recombination C. antigen induction D. clonal selection

Correct Answer: B

In chronic active hepatitis, high titers of which of the following antibodies are seen? A. antimitochondrial B. anti-smooth muscle C. anti-DNA D. anti-parietal cell

Correct Answer: B

In hybridoma technology, the desirable fused cell is the: A. myeloma-myeloma hybrid B. myeloma-lymphocyte hybrid C. lymphocyte-lymphocyte hybrid D. lymphocyte-granulocyte hybrid

Correct Answer: B

In the indirect fluorescent antibody test, a speckled pattern may indicate the presence of antibody to: A. histone B. Sm C. RNA D. DNA

Correct Answer: B

In the interpretation of agglutination tests for febrile disease, which of the following is of the greatest diagnostic importance? A. anamnsetic reactions caused by heterologous antigens B. rise in titer of the patient's serum C. history of previous vaccination D. naturally occurring antibodies prevalent where the disease is endemic

Correct Answer: B

Interpret the following microcytotoxicity results: -A9 and B12 cells damaged -A1 and Aw19 cells intact A. Positive for A1 and Aw19; negative A9 and B12 B. Negative for A1 and Aw19; positive for A9 and B12 C. Error in test system; retest D. Impossible to determine

Correct Answer: B

Interpret the following results for HIV infection. -ELISA: positive -repeat ELISA: negative; -Western blot: no bands A. Positive for HIV B. Negative for HIV C. Indeterminate D. Further testing needed

Correct Answer: B

It is important to note that when an infant is born, levels of specific antibody of the following class are used to indicate neonatal infection: A. IgA B. IgG C. IgM D. IgD

Correct Answer: B

Measurement of serum levels of which of the following immunoglobulins can serve as a screening test for multiple allergies? A. IgA B. IgE C. IgG D. IgM

Correct Answer: B

Potent chemotactic activity is associated with which of the following components of the complement system: A. C1q B. C5a C. C3b D. IgG

Correct Answer: B

Refer to the following data from a peripheral blood sample: total WBC: 10.0 x 10^3/uL Differential: -neutrophils: 68% -lymphocytes: 25% (40% T cells) -monocytes: 4% -eosinophils: 2% -basophils: 1% The expected total number of T cells is: A. 200 B. 1,000 C. 2,000 D. 2,500

Correct Answer: B

Refer to the following data: HBsAg; anti-HBc IgM; anti-HAV IgM patient#1: neg, neg, pos patient#2: pos, pos, neg patient#3: neg, post , neg From the test results above, it can be concluded that patient #3 has: A. recent acute hepatitis A B. acute hepatitis B C. acute hepatitis C (non-A/non-B hepatitis) D. chronic hepatitis B

Correct Answer: B

Refer to the following illustration of the hepatitis B virus: See BOC pg 247 Select the corresponding lettered component indicated on the diagram for viral DNA. A. A B. B C. C D. D

Correct Answer: B

Refer to the following illustration: See BOC pg 238 Pic 3 Which of the above figures demonstrates a reaction pattern of nonidentity? A. Figure #1 B. Figure #2 C. Figure #3 D. Figure #4

Correct Answer: B

SITUATION: A 54-year-old man was admitted to the hospital after having a seizure. Many laboratory test were preformed, including an RPR, but none of the results were positive. The physician suspects a case of late (tertiary) syphilis. Which test should be performed next? A. Repeat RPR, then perform VDRL B. Treponemal test such as MHA-TP on serum C. VDRL on CSF D. No laboratory test is positive for late (tertiary) syphilis

Correct Answer: B

SITUATION: Cells type negative for all HLA antigens in a complement-dependent cytotoxicity assay. What is the most likely cause? A. Too much supravital dye was added B. Rabbit complement is inactivated C. All leukocytes are dead D. Antisera is too concentrated

Correct Answer: B

Sera to be tested for IFA-ANA 6 days after drawing is best stored at: A. room temperature B. 5°C +-2 C C. -70°C in a constant temperature freezer D. -20°C in a frost-free self-defrosting freezer

Correct Answer: B

Serolocial test for which disease may give a false-positive result if the patient has Lyme disease? A. AIDS B. Syphilis C. Cold agglutinins D. Hepatitis C

Correct Answer: B

Soluble immune complexes are formed under the condition of A. antigen deficiency B. antigen excess C. antibody excess D. complement

Correct Answer: B

Substances that are antigenic only when coupled to a protein carrier are: A. opsonins B. haptens C. adjuvants D. allergens

Correct Answer: B

T cells are incapable of: A. collaborating with B cells in antibody responses B. secretion of immunoglobulins C. secretion of cytokines D. producing positive skin tests

Correct Answer: B

Test to identify infection with HIV fall into which three general classification types of test? A. Tissue culture, antigen, and antibody tests B. Tests for antigen, antibodies, and nucleic acid C. DNA probe, DNA amplification, and Western blot tests D. ELISA, Western blot, and Southern blot tests

Correct Answer: B

The FTA-ABS test for the serological diagnosis of syphilis is: A. less sensitive and specific than the VDRL is properly performed B. likely to remain positive after adequate antibiotic therapy C. currently recommended for testing cerebrospinal fluid D. preferred over darkfield microscopy for diagnosing primary syphilis

Correct Answer: B

The air temperature throughout the serology laboratory is 20C. How will this affect VDRL and RPR test results? A. no effect-the acceptable test range is 20-24C B. weaken reactions so that false negatives occur C. strengthen reactions of that positive titers appear elevated D. increase the number of false positives from spontaneous clumping

Correct Answer: B

The antigen marker most closely associated with transmissibility of HBV infection is: A. HBsAg B. HBeAg C. HBcAg D. HBV

Correct Answer: B

The areA of the immunoglobulin molecule referred to as the hinge region is located between which domains? A. V(h) and V(l) B. C(h1) and C(h2) C. C(h2) and C(h3) D. C(h3) and V(l)

Correct Answer: B

The autosomal recessive form of severe combined immunodeficiency disease is also referred to as: A. Bruton agmmaglobulinemia B. Swiss-type lymphopenic agammaglobulinemia C. DiGeorge syndrome D. Wiskott-Aldrich syndrome

Correct Answer: B

The complement component C3: A. is increased (in plasma levels) when complement activation occurs B. can be measure by immunoprecipitin assays C. causes the conversion of C4 to C4a + C4b D. is not involved in the alternate complement pathway

Correct Answer: B

The curve below was obtained by adding increaseing amounts of a soluble antigen to fixed volumes of monospecific antiserum: See BOC pg 236 Pic 2 The area on the curve where no precipitate formed due to antigen excess is: A. A B. B C. C D. D

Correct Answer: B

The hyperviscosity syndrome is most likely to be seen in monoclonal disease of which of the following immunoglobulin classes? A. IgA B. IgM C. IgG D. IgD

Correct Answer: B

The immunoglobulin class typically found to be present in saliva, tears, and other secretions is: A. IgG B. IgA C. IgM D. IgD

Correct Answer: B

The most rapid immediate hypersentivity reaction is associate with: A. transfusion B. anaphylaxis C. contact dermatitis D. serum sickness

Correct Answer: B

The ratio of kappa to lambda light chain producing cells in normal individuals is: A. 1:1 B. 2:1 C. 3:1 D. 4:1

Correct Answer: B

The serological test for syphilis recommended for detecting antibody in cerebrospinal fluid is: A. nontreponemal antibody B. CSF-VDRL C. FTA-ABS D. MHA-TP

Correct Answer: B

The serological test that can be modified to selectively detect only specific IgM antibody in untreated serum is: A. Ouchterlony B. enzyme immunoassay C. hemagglutination inhibition D. passive hemagglutination

Correct Answer: B

The technologist observes apparent homogenous staining of the nucleus of interphase cells while performing an IFA-ANA, as well as staining of the chromosomes in mitotic cells. This result is: A. indicative of 2 antibodies, which should be separately reported after titration B. expected for anti-DNA antibodies C. inconsistent; the test should be reported with new reagent D. expected for anti-centromere antibodies

Correct Answer: B

The visible serological reaction between soluble antigen and its specific antibody is: A. sensitization B. precipitation C. agglutination D. opsonization

Correct Answer: B

Treatment of IgG with papain results in how many fragments from each immunoglobulin molecule? A. 2 B. 3 C. 4 D. 5

Correct Answer: B

What affect does selecting the wrong gate have on the results when cells are counted by flow cytometry? A. No effect B. Failure to count the desired cell population C. Falsely elevated results D. Impossible to determine

Correct Answer: B

What assay would confirm the immune status to hepatitis B virus? A. HBsAg B. anti-HBs C. IgM anti-HBcAg D. hepatitis C Ag

Correct Answer: B

What constitutes a diagnosis of viral hepatitis? A. Abnormal test results for liver enzymes B. Clinical sign and symptoms C. Positive results for hepatitis markers D. All of these options

Correct Answer: B

What immune elements are involved in a positive skin test for tuberculosis? A. IgE antibodies B. T cells and macrophages C. NK cells and IgG antibody D. B cells and IgM antibody

Correct Answer: B

What is a general definition for autoimmunity? A. Increase of tolerance to self-antigens B. Loss of tolerance to self-antigens C. Increase in clonal deletion of mutant cells D. Manifestation of immunosuppression

Correct Answer: B

What is the endpoint for the antistreptolysin O (ASO) latex agglutination assay? A.Highest serum dilution that show no agglutination B. Highest serum dilution that shows agglutination C. Lowest serum dilution that shows agglutination D. Lowest serum dilution that show no agglutination

Correct Answer: B

What is the main advantage of the recovery and reinfusion of autologous stem cells? A. It slows the rate of rejection of transplanted cells B. It prevents graft-versus-host disease C. No HLA testing is required D. Engraftment occurs in a more efficient sequence

Correct Answer: B

What is the most likely cause when a Western blot or ELISA is positive for all controls and samples? A. Improper pipetting B. Improper washing C. Improper addition of sample D. Improper reading

Correct Answer: B

What is the most likely explanation when antibody test for HIV are negative but a polymerase chain reaction test performed 1 week later is positive? A. Probably not HIV infection B. Patient is in the "window phase" before antibody production C. Test were performed incorrectly D. Clinical signs may be misinterpreted

Correct Answer: B

What is the titer in tube No. 8 if tube No. 1 is undiluted and dilutions are doubled? A. 64 B. 128 C. 256 D. 512

Correct Answer: B

What outcome results form improper washing of a tube or a well after adding the enzyme-antibody conjugate in an ELISA system? A. Result will be falsely decreased B. Result will be falsely increased C. Result will be unaffected D. Result is impossible to determine

Correct Answer: B

What substance is detected by the rapid plasma reagin (RPR) and Venereal Disease Research Laboratory (VDRL) test for syphilis? A. Cardiolipin B. Anticardiolipin antibody C. Anti-T. pallidum antibody D. Treponema pallidum

Correct Answer: B

What would happen if the color reaction phase is prolonged in one tube or well of an ELISA test? A. Result will be falsely decreased B. Result will be falsely increased C. Result will be unaffected D. Impossible to determine

Correct Answer: B

When testing a patient for HIV antibody, which of the following is used to confirm a positive screening test? A. radioimmunoassay B. Western blot C. immunofluorescence D. ELISA

Correct Answer: B

Which antibody persist in low-level carriers of hepatitis B virus? A. IgM anti-HBc B. IgG anti-HBc C. IgM anti-HBe D. IgG anti-HBs

Correct Answer: B

Which control shows the correct result for a valid ASO test? A. SLO control, no hemolysis B. Red cell control, no hemolysis C. Positive control, hemolysis in all tubes D. Hemolysis in both SLO and red cell control

Correct Answer: B

Which disease might be indicated by antibodies to smooth muscle? A. Atrophic gastritis B. Autoimmune hepatitis C. Myasthenia gravis D. Sjogren's syndrome

Correct Answer: B

Which is the first antibody detected in serum after infection with hepatitis B virus (HBV)? A. Anti-HBs B. Anti-HBc IgM C. Anti-HBe D. All are detectable at the same time

Correct Answer: B

Which method, classically used for HLA-D typing, is often used to determine the compatibility between a living organ donor and recipient? A. Flow cytometry B. Mixed lymphocyte culture (MLC) C. Primed lymphocyte test (PLT) D. Restriction fragment length polymorphism (RELP)

Correct Answer: B

Which of the following fungal organisms is best diagnosed by an antibody detection test as opposed to an antibody detection assay? A. Histoplasma B. Cryptococcus C. Candida D. Aspergillus

Correct Answer: B

Which of the following immunoglobulins is present is the highest concentration in normal human serum? A. IgM B. IgG C. IgA D. IgE

Correct Answer: B

Which of the following is an important cellular mediator immune complex tissue injury? A. mast cell B. neutrophil C. basophil D. eosinophil

Correct Answer: B

Which of the following is most useful in establishing a diagnosis in the convalescence phase of a viral infection? A. slide culture B. serological techniques C. shell vial D. culture of McCoy media

Correct Answer: B

Which of the following is the best indicator of an acute infection with the hepatitis A virus? A. the presence of IgG antibodies to hepatitis A virus B. the presence of IgM antibodies to hepatitis A virus C. a sharp decline in the level of IgG antibodies to hepatitis A virus D. a rise in both IgM and IgG levels of antibody to hepatitis A virus

Correct Answer: B

Which of the following is the larger residual split portion of C3? A. C3a B. C3b C. C4 D. C1q

Correct Answer: B

87. A living donor is being sought for a child who requires a kidney transplant. The best odds of finding an MHC-compatible donor occur between the child and A. A sibling (brother or sister) B. An unrelated individual C. The child's father D. The child's mother

Correct Answer: A Because the human leukocyte antigen (HLA) system is extremely polymorphic, the odds are greatly against finding an HLA-compatible donor in unrelated individuals. The genes coding for HLA antigens are inherited from one's parents and are expressed co-dominantly. Between an offspring and either parent, there is, statistically, a 25% chance of an HLA match. Between siblings, there is a 50% chance of an HLA match

Which of the following is used in rapid slide test for detection of rheumatoid factors? A. Whole IgM molecules B. Fc portion of the IgG molecule C. Fab portion of the IgG molecules D. Fc portion of the IgM molecules

Correct Answer: B

Which of the following positive antibody tests may be an indication of recent vaccination or early primary infection for rubella in a patient with no clinical symptoms? A. Only IgG antibodies positive B. Only IgM antibodies positive C. Both IgG and IgM antibodies positive D. Fourfold rise in titer for IgG antibodies

Correct Answer: B

Which serum antibody response usually characterized the primary (early) stage of syphilis? A. Antibodies against syphilis are undetectable B. Detected 1-3 weeks after appearance of the primary chancre C. Detected in 50% of cases before the primary chancre disappears D. Detected within 2 weeks after infection

Correct Answer: B

Which test are considered confirmatory test for HIV? A. ELISA and rapid antibody tests B. Western blot test, HIC-1,2 differentiation assays, and polymerase chain reaction C. Culture, antigen capture assay, polymerase chain reaction D. Reverse transcriptase and mRNA assay

Correct Answer: B

Which test detects antibodies that have attached to tissues, resulting in a type-II cytotoxic reaction? A. Migration inhibition factor assay (MIF) B. Direct immunofluorescence (IF) C. Immunofixation electrophoresis (IFE) D. Hemagglutination

Correct Answer: B

Which test would measure the coating of red cells by antibody as occurs in hemolytic transfusion reactions? A. Indirect antiglobulin test (IAT) B. Direct antiglobulin test (DAT) C. ELISA D. Hemagglutination

Correct Answer: B

Which test, other than serological markers, is most consistently elevated in viral hepatitis? A. Antinuclear antibodies B. Alanine aminotransferase (ALT) C. Absolute lymphocyte count D. Lactate dehydrogenase

Correct Answer: B

Why is skin testing the most widely used method to test for a type I hypersensitivity reaction? A. It causes less trauma and ism ore cost effective than other methods B. It has greater sensitivity than in vitro measurements C. It is more likely to be positive for IgE-specific allergens than other methods D. It may be used to predict the development of further allergen sensitivity

Correct Answer: B

Would an hCG test using a monoclonal antibody against the B-subunit of hCG likely be affected by an increased level of follicle-stimulating hormone (FSH)? A. Yes, the B-subunit of FSH is identical to that of hCG B. NO, the test would be specific for the B-subunit of hCG C. Yes, a cross reaction would occur because of structural similarities D. No, the structure of FSH an hCG are not all similar

Correct Answer: B

6. Cell-mediated immunity is primarily mediated by: A. B cells B. T helper cells C. Plasma cells D. Dendritic cells

Correct Answer: B T helper cells are the primary mediators of cell-mediated immunity (CMI). They secrete several different lymphokines that stimulate a number of other cells, such as cytotoxic T lymphocytes and monocytes. B cells differentiate into plasma cells during a humoral-mediated immune response. Dendritic cells are important antigen presenting cells, but they are not the primary mediators of a CMI response.

36. The strength with which a multivalent antibody binds a multivalent antigen is termed the A. Affinity B. Avidity C. Reactivity D. Valence

Correct Answer: B "Avidity" is used to describe the strength of binding between a multivalent antibody and multivalent antigen. "Affinity" describes the bond between a single antigenic determinant and an individual combining site. "Valence" refers to the number of antigenic determinants on an antigen.

64. The primary mechanism responsible for pathology in systemic lupus erythematosus is A. Allergic reaction to foreign molecules B. Antibodies directed against self antigens C. Polyclonal activation of cytotoxic T cells D. Lack of intracellular killing after neutrophils phagocytosis of bacteria

Correct Answer: B Antibodies directed against self antigens form immune complexes and activate complement. Circulating immune complexes, composed of nuclear antigen and antinuclear antibody, deposit in various organ systems, activate complement, and produce organ pathology. T cells are not directly involved in this process. Allergens, phagocytosis, and killing of ingested bacteria by neutrophils do not play a role in the pathogenic process.

71. Which of the following statements about the test for C-reactive protein (CRP) is true? A. It correlates with neutrophil phagocytic function B. It is an indicator of ongoing inflammation C. It is diagnostic for rheumatic fever D. Levels decrease during heart disease

Correct Answer: B CRP is an acute-phase reactant. Although it is elevated in inflammation, its presence is not diagnostic for any one disease, such as rheumatic fever. It does not correlate with antibody levels or with neutrophil phagocytic function. CRP levels are sometimes elevated during heart disease.

92. Contact dermatitis is mediated by A. B lymphocytes B. T lymphocytes C. Macrophages D. Polymorphonuclear cells

Correct Answer: B Contact dermatitis is a delayed-type hypersensitivity reaction mediated by T cells. Antibody is not involved in this type of hypersensitivity, so B cells play no role in it. Neither macrophages nor neutrophils are involved in this type of hypersensitivity.

84. Rheumatic fever sometimes occurs after group A streptococcal infections. in this condition, an autoimmune response attacks the tissue of the heart valves. This phenomenon is an example of A. Epitope spreading B. Molecular mimicry C. Polyclonal B cell activation D. Preferential activation of T helper cells

Correct Answer: B Group A streptococci contain antigenic determinants that are similar to antigenic determinants found on heart valve tissue in some individuals. The immune response occurring during the course of a group A streptococcal infection may be extensive enough to include an immune-mediated attack on the heart valves— rheumatic heart disease. "Molecular mimicry" is the term given to this phenomenon, whereby an immune response directed against one antigen may be extended to include activity against closely related antigens.

53. Elevated IgE levels are typically found in A. Type I hypersensitivity reactions B. Type II hypersensitivity reactions C. Type III hypersensitivity reactions D. Type IV hypersensitivity reactions

Correct Answer: A Elevated IgE levels are found in type I hypersensitivity reactions. The antibody binds via the Fc portion of the molecule to Fc receptors on mast cells and basophils. When the attached antibody binds its specific allergen, the cell degranulates.

8. HLA antigens are found on A. All nucleated cells B. Red blood cells only C. Solid tissue only D. Whit blood cells only

Correct Answer: A Human leukocyte antigens (HLAs) are a group of antigens originally described on human white cells. It is now known that they are found on all nucleated cells of the body, including solid tissue cells. HLAs are not found on red blood cells.

26. In individuals allergic to pollen, hyposensitization protocols may be initiated. These individuals receive injections of A. Allergen B. Pooled human antisera C. Monoclonal antibody directed against human T cells D. Monoclonal antibody directed against human B cells

Correct Answer: A Hyponsensitization, allergy injections, involves the administration of gradually increasing concentrations of an allergen. The goal is for the patient to become tolerant of the allergen and no longer exhibit an allergic response to the allergen. It is hypothesized that patients will ultimately develop high concentrations of IgG to the allergen, blocking IgE from binding and thereby preventing the allergic reaction.

16. Color Plate 22 represents a dimeric IgA molecule. The structure printed in red and indicated by the red arrow is called the A. J-piece B. Hinge region C. Heavy chain D. Light chain

Correct Answer: A IgA is found in mucous secretions as a dimer stabilized by the J-piece. IgA is synthesized locally by plasma cells and dimerized intracellularly. IgM is also held together by a J-piece, but il exists as a pentamer.

95. In a chemiluminescent immunologic assay, what is the signal detected? A. Light B. An electric signal C. A purple-colored compound D. A yellow-colored compound

Correct Answer: A In chemiluminiscent assays, light is the end product. These assays require special instruments to measure the light emitted in the reaction. Chemicals used to generate light include luminol and luciferase.

50. Which of the following serologic tests is commonly performed by an immunofluorescence method? A. Anti-HBS B. Antinuclear antibody (ANA) C. Antistreptolysin O (ASO) D. C-reactive protein (CRP)

Correct Answer: B Testing for antinuclear antibodies (ANAs) is commonly performed by the immunofluorescence method—using fluorescein-conjugated antihuman antibody to detect patient antibody bound to nuclear components of test cells. Anti-streptolysin O tests are performed with red cells or, more recently, by latex agglutination. Anti-HBs assays are generally performed by ELISA, and C-reactive protein assays are generally performed by latex agglutination, turbidimetry, or nephelometry.

7. The HLA complex is located primarily on A. Chromosome 3 B. Chromsome 6 C. Chromosome 9 D. Chromosome 17

Correct Answer: B The HLA system is part of a larger region known as the major histocompatibility complex. It is located on chromosome 6. The region is located on the short arm of the chromosome. Chromosome 15 contains one HLA gene, B2M

45. The Venereal Disease Research Laboratory (VDRL) test for syphilis is classified as a(n) A. Agglutination reaction B. Flocculation reaction C. Hemagglutination reaction D. Precipitation reaction

Correct Answer: B The cardiolipin antigen is particulate, not soluble, in the VDRL test. However, the particles are too small to make macroscopic agglutinates when combined with antibody. This type of reaction is called a flocculation reaction and needs to be read with low-power microscopy.

38. After activation of the complement system, leukocytes and macrophages are attracted to the site of complement activation by A. C1 B. C5a C. C8 D. IgM

Correct Answer: B The complement-activation product C5a is chemotactic for neutrophils and macrophages. Neither C1 nor C8 (which occur in the plasma before complement activation) possesses such chemotactic properties. IgM antibody, although capable of activating complement by the classical pathway, is not a chemotactic factor for phagocytic cells.

14. Which class of immunoglobulin possesses delta heavy chains? A. IgA B. IgD C. IgE D. IgG

Correct Answer: B The heavy chains divide human immunoglobulin molecules into separate classes and subclasses. The della heavy chain corresponds to IgD. The remaining classes IgA, IgE, IgG, and IgM correspond to α, ε, γ, and μ, respectively.

52. In the enzyme-linked immunosorbent assay (ELISA), the visible reaction is due to a reaction between: A. Enzyme and antibody B. Enzyme and substrate C. Fluorescent dye and antigen D. Latex particles and antibody

Correct Answer: B The indicator system in an ELISA test con- sists of an enzyme and its substrate. If the enzyme-labeled antibody has complexed with the immobilized antigen, the addition of substrate will produce a colored end product. Alkaline phosphatase is an enzyme frequently used in ELISA tests. Latex particles, fluorescent dyes, and red blood cells are not used in ELISA tests but in other test methodologies.

31. Diagnosis of group A streptococci (Streptococcus pyogenes) infection is indicated by the presence of: A. Anti-protein A B. Anti-DNase B C. Anti-beta-toxin D. C-reactive protein

Correct Answer: B The serological diagnosis of group A streptococcal infection can be made by demonstrating anti-DNase B. The antistreptolysin O (ASO) assay can also be used; however, ASO response is poor in skin infections. C-reactive protein is an acute-phase protein indicating inflammation.

49. IgM antibodies react well in complement fixation (CF) tests. Because of this, CF test for antibodies should A. Be positive early in the course of the disease B. Be useful in identifying antibodies responsible for a delayed hypersensitivity reaction C. Be useful in identifying antibodies responsible for anaphylactic reactions D. Detect transplacental antibodies

Correct Answer: A In most infections, IgM antibodies will develop first followed by IgG, which develop higher titers and are longer lasting. Anaphylactic reactions are caused by IgE antibody. Delayed hypersensitivity reactions are caused by T cells. Transplacental antibodies belong to the IgG class. IgG antibodies, although they can be detected by complement fixation (CF), do not fix complement efficiently

35. A major advantage of passive immunization compared to active immunization is that: A. Antibody is available more quickly B. Antibody persists for the life of the recipient C. IgM is the predominant antibody class provided D. Oral administration can be used

Correct Answer: A In passive immunization, preformed antibody is delivered to the recipient, making the antibody available immediately. In active immunization, a period of days is required before antibody production occurs. Passive immunity is short lived, in contrast to the possibly lifelong persistence of actively induced antibody. Because passive immunization involves the transfer of antibodies, the oral route cannot be used—antibodies are digested in the gastrointestinaltract. The antibodies administered by passive immunization consist largely of the IgG class.

A patient has a prostate-specific antigen level of 60 ng/mL the day before surgery to remove a localized prostate tumor. One week following surgery, the serum PSA was determined to be 8 ng/mL by the same method. What is the most likely cause of these results? A. Incomplete removal of the malignancy B. Cross reactivity of the antibody with another tumor antigen C. Testing too soon after surgery D. Hook effect with the PSA assay

Correct Answer: C

A patient has the following test results: ANA ; ASO ; Complement ; RA +, 1:320; 50 Todd units; decreased; + The above results could be seen in patients with: A. rheumatic fever B. rheumatoid arthritis C. lupus erythematosus D. glomerulonephritis

Correct Answer: C

A patient received 2 units of RBCs following surgery. Two weeks after the surgery, the patient was seen by his physician and exhibited mild jaundice and slightly elevated liver enzymes. Hepatitis testing, however, was negative. What should be done next? A. Nothing until more sever or definitive clinical signs develop B. Repeat hepatitis testing immediately C. Repeat hepatitis testing in a few weeks D. Check blood bank donor records and contact donor(s) of transfused units

Correct Answer: C

5. Monocytes and macrophages play a major role in the mononuclear phagocytic system. For an antibody-coated antigen to be phagocytized, what part of the antibody molecule fits into a receptor on the phagocytic cell? A. Fc region B. Fab region C. Hinge region D. Variable region

Correct Answer: A The Fc region of an IgG molecule fits into an Fc receptor (FcR) on macrophages and monocytes. The Fc receptor binds to specific amino acid residues in the Fc region of the immunoglobulin. The variable region of immunoglobulin binds to the antigen.

73. The alternative complement pathway A. Can be activated by bacterial capsule polysaccharides B. Uses C5b as a C3 convertase C. Bypasses steps C3 through C5 D. Is activated by properdin

Correct Answer: A The alternative pathway for complement activation is a more nonspecific defense mechanism, in that it does not require the presence of antibody for activation. It can be activated by a variety of substances, including complex polysaccharides found in bacterial capsules and cell walls. These materials activate C3 directly. Properdin protein stabilizes some of the active complement proteins, and C4b2a is a C3 convertase.

A patient received 5 units of fresh frozen plasma (FFP) and developed a severe anaphylactic reaction. He has a history of respiratory and gastrointestinal infections. Post-transfusion studies showed all 5 units to be ABO-compatible. What immunologic test would help to determine the cause of this transfusion reaction? A. Complement levels, particularly C3 and C4 B. Flow cytometry for T-cell counts C. Measurement of immunoglobulins D. NBT test for phagocytic function

Correct Answer: C

1. Color Plate 21 depicts a monomeric immunoglobulin molecule. The portion of the molecule indicated by the dotted red circle and the red arrow is called the A. Fab fragment B. Fc fragment C. Heavy chain D. Hinge region

Correct Answer: A The basic structure of all immunoglobulins is two light chains joined to two heavy chains by disulfide bonds. The amino terminus of both the heavy and light chains, together, constitutes the Fab fragment (fragment of antigen binding). The carboxy-terminus of the heavy chains constitutes the Fc fragment. The hinge region is the area at the center of the "Y," near the carboxy- terminus of the light chains.

A positive ANA with the pattern of anticentromere antibodies is most frequently seen in patients with A. rheumatoid arthritis B. systemic lupus erythematosus C. CREST syndrome D. Sjorgren sysdrome

Correct Answer: C

A substrate is first exposed to a patient's serum, the after washing, anti-human immunoglobulin labeled with a fluorochrome is added. The procedure described is: A. fluorescent quenching B. direct fluorescence C. indirect fluorescence D. fluorescence inhibition

Correct Answer: C

A woman who has and five pregnancies subsequently tests positive for HIV by Western blot. What is the most likely reason for this result? A. Possible cross-reaction with herpes or EBV antibodies B. Interference form medication C. Cross-reaction with HLA antigens in the antigen preparation D. Possible technical error

Correct Answer: C

A woman who has been pregnant for 12 weeks is tested for toxoplasmosis. Her IgM ELISA titer is 2.6 (reference range < 1.6), and her IgG ELISA value is 66 (reference range < 8). The physician asks you if these results indicated an infection during the past 12 weeks. Which of the following test would you recommend to determine if the woman was infected during her pregnancy? A. Toxo PCR on amniotic fluid B. Toxo IgM on amniotic fluid C. Toxo IgG avidity D. Amniotic fluid culture

Correct Answer: C

All of the following hepatitis viruses are spread through blood or blood products except: A. Hepatitis A B. Hepatitis B C. Hepatitis C D. Hepatitis D

Correct Answer: C

All positive cold agglutinin tubes remain positive after 37C incubation except the positive control. What is the most likely explanation for this situation? A. High titer cold agglutinins B. Contamination of the test system C. Antibody other than cold agglutinins D. Faulty water bath

Correct Answer: C

An antinuclear antibody test is performed on a specimen form a 55-year-old woman who has unexplained joint pain. The IFA result is a titer of 40 and a homogeneous pattern. The appropriate follow-up for this patient is: A. Anti-DNA assay B. Extractable nuclear antigen (ENA) testing C. Retest ANA in 3-6 months D. CH50 complement assay

Correct Answer: C

An example of an organ specific disease with disease with autoimmune antibodies is: A. Wegener granulomatosus B. rheumatoid arthritis C. Hashimoto thyroiditis D. systemic lupus erythematosus

Correct Answer: C

An immunosuppressed patient has an unexplained anemia. The physician suspects a parvovirus B19 infection. A parovirus IgM test is negative. The next course of action is to tell the physician: A. The patient does not have parovirus B. A convalescent specimen is recommended in 4 weeks to determine if a fourfold rise in titer has occurred C. A parovirus PCR is recommended D. That a recent transfusion for the patient's anemia may have resulted in a false-negative assay and the patient should be retested in 4 weeks

Correct Answer: C

An initial and repeat ELISA test for antibodies to HIV-1 are both positive. A Western blot shows a single band at gp160. The patient shows no clinical signs of HIV infection, and the patient's CD4 T-cell count is normal. Base upon these results, which conclusion is correct? A. Patients is diagnosed as HIV-1-positive B. Patients is diagnose as HIV-2 positive C. Results are inconclusive D. Patient is diagnosed as HIV-1 negative

Correct Answer: C

Antibodies composed of IgG immunoglobin: A. occur during the primary response to antigen B. are larger molecules than IgM antibodies C. can cross the placenta from mother to fetus D. can be detected in saline crossmatches

Correct Answer: C

Antibody idiotype is dictated by the: A. constant region of heavy chain B. constant region of light chain C. variable regions of heavy and light chains D. constant regions of heavy and light chains

Correct Answer: C

Blood products are tested for which virus before being transfused to newborns? A. EBV B. Human T-lymphotropic virus II (HTLV-II) C. Cytomegalovirus (CMV) D. Hepatitis D virus

Correct Answer: C

C3b and Fc receptors are present on: A. B lymphocytes B. monocytes C. B lymphocytes and monocytes D. neither B lymphocytes and monocytes

Correct Answer: C

Combined immunodeficiency disease with loss of muscle coordination is referred to as: A. DiGeorge syndrome B. Bruton agammaglobulinemia C. ataxia telangiectasia D. Wiskott-Aldrich syndrom

Correct Answer: C

Flocculation test for syphilis use antigen composed of: A. Treponema pallidum B. reagin C. cardiolipin and lecithin D. charcoal

Correct Answer: C

For diagnosis of late latent of tertiary syphilis, the most appropriate assay is: A. RPR B. VDRL C. FTA-ABS D. FTA-ABS IgM

Correct Answer: C

For several months a 31-year-old woman has had migratory polyarthritis and a skin rash. Upon admission to the hospital, the following laboratory data were obtained: Patient ; Reference range leukocyte count: 4.7 x 10^3/uL ; 5.0-10.0 x 10^3/uL differential: normal serum hemolytic complement: <22U ANA: positive in a homogenous pattern rheumatoid factor test: negative urinalysis: protein 1+, occasional RBCs This patient's test results are consistent with: A. dermatomyositis B. C1INH deficiency C. systemic lupus erythematosus D. mixed connective tissue disease

Correct Answer: C

Free monoclonal light chains are often present in the serum of multiple myeloma patients, and may be useful for disease monitoring. Which of the following assays would b recommended to detect the presence of serum-free light chain. A. Serum protein electrophoresis B. Urine immunofixation C. Nephelometry D. ELISA

Correct Answer: C

Give this hematologic data: WBC: 5.0 x 10^3/uL Lymphs: 15% CD4: 8% Which of the following is the correct interpretation: A. CD4% and absolute CD4 normal B. consistent with an intact immune system C. consistent with a viral infection such as HIV D. technical error

Correct Answer: C

Given a heterophile antibody titer of 224, which of the following results indicate serum sickness? Absorption with Guinea Pig Kidney; Absorption with Beef Cells A. Two-tube titer reduction; Five-tube titer reduction B. No titer reduction; No titer reduction C. Five-tube titer reduction; Five-tube titer reduction D. Five-tube titer reduction; No titer reduction

Correct Answer: C

Goat anti-human IgG heavy chain specific alkaline phosphatase conjugate is a: A. monoclonal reagent that reacts with gamma heavy chains B. monoclonal reagent that reacts with light chains C. polyclonal reagent that reacts with gamma heavy chains D. polyclonal reagent that reacts with light chains

Correct Answer: C

HLA-B8 antigne has been associated with which of the following pairs of disease? A. ankylosing spondylitis and myasthenia gravis B. celiac disease and ankylosing spondylitis C. myasthenia gravis and celiac disease D. Reiter disease and multiple sclerosis

Correct Answer: C

Hepatitis C differs from hepatitis A because it: A. has a highly stable incubation period B. is associated with a high incidence of icteric hepatitis C. is associated with a high incidence of the chronic carrier state D. is seldom implicated in cases of posttransfusion hepatitis

Correct Answer: C

Hereditary deficiency of early complement components (C1, C4, and C2) is associated with: A. pneumococcal septicemia B. small bowel obstruction C. lupus erythematosus like syndrome D. gonococcemia

Correct Answer: C

Humoral antibodies are produced by which cells? A. macrophages B. T lymphocytes C. B lymphocytes D. neutrophils

Correct Answer: C

In an anti-nuclear antibody indirect immunofluorescence test, a sample of patient serum shows a positive, speckled pattern. Which would be the most appropriate additional test to perform? A. antimitochondrial antibody B. immunoglobulin quantitation C. screen for Sm and RNP antibodies D. anti-DNA antibody using C luciliae

Correct Answer: C

In flow cytometry, labeled cells: A. scatter the light absorb fluorescence B. absorb fluorescence and emit electronic impulses C. scatter the light and emit fluorescence D. absorb both fluorescence and light

Correct Answer: C

In monitoring an HIV-infected patient, which parameter may be expected to be the most sensitive indicator of the effectiveness of antiretroviral treatment? A. HIV antibody titer B. CD4:CD8 ratio C. HIV viral load D. Absolute total T-cell count

Correct Answer: C

In the anti-double-stranded DNA procedure, the antigen most commonly utilized is: A. rat stomach tissue B. Mouse kidney tissue C. Crithidia luciliae D. Toxoplasma gondii

Correct Answer: C

In the indirect immunofluorescence method of antibody detection in patient serum, the labeled antibody is: A. human anti-goat immunoglobulin B. rheumatoid factor C. goat anti-human immunoglobulin D. complement

Correct Answer: C

93. In a competitive radiomimmunosorbent test (RIST), what does a high signal suggest? A. The patient sample has a low concentration of IgE B. The patient sample has a low concentration of IgM C. The patient sample has a high concentration of IgE D. The patient sample has a high concentration of total antibody

Correct Answer: A The competitive RIST assay is used to determine the concentration of total IgE. Patient sample containing IgE is mixed with labeled IgE. Both labeled and unlabeled IgE are captured by antihuman IgE. After a wash step, the signal from the label is detected. A high signal indicates a low concentration of unlabeled IgE from the patient sample.

Inflammation involves a variety of biochemical and cellular mediators. Which of the following may be increased within 72 hours after an initial infection? A. Neutrophils, macrophages, antibody, complement, a1-antitrypsin B. Macrophages, T cells antibody, haptoglobin, fibrinogen C. Neutrophils, macrophages, complement, fibrinogen, C-reactive protein D. Macrophages, T cells, B cells, ceruloplasmin, complement

Correct Answer: C

Interpret the following description of an immunofixation electrophoresis assay of urine. Dense wide bands in both the k and (greek h) lanes. No bands present in the heavy-chain lanes. A. Normal B. Light chain disease C. Increased polyclonal Fab fragments D. Multiple myeloma

Correct Answer: C

Interpret the following results for EBV infection: IgG and IgM antibodies to viral capsid antigen (VCA) are positive. A. Infection in the past B. Infection with a mutual enhancer virus such as HIV C. Current infection D. Impossible to interpret: need more information

Correct Answer: C

Membrane-bound immunoglobulin molecules: A. have an additional amino-terminal sequence of about 40 residues B. are an are not anchored in a transmembrane configuration C. are anchored by a hydrophobic sequence of about 26 residues D. are anchored by a hydrophilic region

Correct Answer: C

Nonspecific killing of tumor cells is carried out by: A. cytotoxic T cells B. helper T cells C. natural killer cells D. antibody and complement

Correct Answer: C

Patients suffering from Waldenstrom macroglobulinemia demonstrate excessively increased concentrations of which of the following? A. IgG B. IgA C. IgM D. IgD

Correct Answer: C

Refer to the following illustration See BOC pg 239 Pic 1 Which of the above figures demonstrates a reaction pattern showing 2 different antigenic molecular species? A. Figure #1 B. Figure #2 C. Figure #3 D. Figure #4

Correct Answer: C

Refer to the following illustration of the hepatitis B virus: See BOC pg 246 Pic 3 Select the corresponding lettered component indicated on the diagram for core antigen A. A B. B C. C D. D

Correct Answer: C

Refer to the following illustration: See BOC pg 229 Which of the ANA patterns shown above would be associated with high titers of antibodies to the Sm antigen A. diagram A B. diagram B C. diagram C D. diagram D

Correct Answer: C

Rheumatoid factors are defined as: A. antigens found in the sera of patients with rheumatoid arthritis B. identical to the rheumatoid arthritis preciptin C. autoantibodies with specificity for the Fc portion of the immunoglobulin (IgG) molecule D. capable of forming circulating immune complexes only when IgM-type autoantibody is present

Correct Answer: C

SITUATION: A patient with RA has acute pneumonia but a negative throat culture. The physician suspects an infection with Mycoplasma pneumoniae and requests an IgM-specific antibody test. The test is performed directly on serial dilutions of serum less than 4 hours old. The result is positive, giving a titer of 1:32. However, the test is repeated 3 weeks later, and the titer remains at 1:32. What best explains? A. IgM-specific antibodies do not increase fourfold between acute and convalescent serum B. The results are not significant because the initial titer was not accompanied by a positive test for cold agglutinins C. Rheumatoid factor caused a false-positive test result D. Insufficient time had elapsed between measurement of acute and convalescent samples

Correct Answer: C

Systemic lupus erythematosus patients with active disease often have which of the following test results? A. high titers of antimicrosomal antibodies B. high titers of anti-smooth muscle anitbodies C. marked decrease in serum CH50 D. decreased serum immunoglobulin levels

Correct Answer: C

T lymphocytes are incapable of functioning as: A. cytotoxic cells B. helper cells C. phagocytic cells D. regulatory cells

Correct Answer: C

T lymphocytes that posses the CD8 surface marker mediate which of the following T-cell functions? A. delayed type hypersensitivity B. regulatory C. cytotoxic D. helper

Correct Answer: C

The 20-nm spheres and filamentous structures of HBV are: A. infectious B. circulating aggregates of HBcAg C. circulating aggregates of HBsAg D. highly infectious when present in great abundance

Correct Answer: C

The C3b component of complement: A. is undetectable in pathological sera B. is a component of the C3 cleaving enzyme of the classical pathway C. is cleaved by C3 inactivator into C3c and C4d D. is not part of the alternative pathway

Correct Answer: C

The IIF staining pattern on ethanol-fixed leukocytes slides shows a perinuclear or nuclear staining pattern. This pattern is typically due to: A. C-ANCA B. LKM C. P-ANCA D. GBM

Correct Answer: C

The component associated only with the alternative pathway of complement activation is: A. C4 B. C1q C. properdin factor B D. C3a

Correct Answer: C

The curve below was obtained by adding increasing amounts of a soluble antigen to fixed volumes of monospecific antiserum: See BOC pg 236 Pic 1 The area on the curve for equivalence precipitate is: A. A B. B C. C D. D

Correct Answer: C

The disappearance of HBsAg and HBeAg, the persistence of anti-HBc, the appearance of anti-HBs, and often of anti-HBe indicate: A. early acute HBV hepatitis B. early convalescent phase HBV hepatitis C. recovery phase of acute HBV hepatitis D. carrier state of acute HBV hepatitis

Correct Answer: C

The enzyme control tube in an ASO hemolytic assay exhibits no cell lysis. What is the most likely explanation for this? A. incorrect pH of buffer B. low ionic strength buffer C. oxidation of the enzyme D. reduction of the enzyme

Correct Answer: C

The enzyme-linked immunosorbent assay (ELISA) technique for the detection of HBsAg: A. requires radiolabeled C1q B. is quantitated by degree of fluorescence C. uses anti-HBs linked to horseradish peroxidase D. uses beads coated with HBsAg

Correct Answer: C

The following pattern of agglutination was observed in an antibody titration: Tube#: 1; 2; 3; 4; 5; 6; 7; 8; 9; 10; 11 1+; 2+; 4+; 4+; 3+; 3+; 2+; 1+; 1+; 0; 0 This set of reactions most likely resulted from: A. faulty pipetting technique B. postzoning C. prozoning D. the presence of a high-titer, low-avidity antibody

Correct Answer: C

The immunoglobulin classes most commonly found on the surface of circulating B lymphocytes in the peripheral blood of normal persons are: A. IgM, IgA B. IgM, IgG C. IgM, IgD D. IgM, IgE

Correct Answer: C

The initial immune response following fetal infection with rubella is the production of which class)es of antibodies? A. IgG B. IgA C. IgM D. both IgG and IgA

Correct Answer: C

The key structural difference that distinguishes immunoglobulin subclasses is the: A. number of domains B. stereometry of the hypervariable region C. the sequence of the constant regions D. covalent linkage of the light chains

Correct Answer: C

The most common label in direct fluorescent antibody technique (DFA) is: A. alkaline phosphatase B. horseradish peroxidase C. fluorescein isothiocyanate D. calcofluor white

Correct Answer: C

The most important use of a nontreponemal antibody (NTA) test alone is in: A. establishing the diagnosis of acute active syphilis B. establishing the diagnosis of chronic syphilis C. evaluating the success of therapy D. determining the prevalence of disease in the general population

Correct Answer: C

The specificity of an immunoassay is determined by the: A. label used on the antigen B. method used to separate the bound form free antigen C. antibody used in the assay D. concentration of unlabeled antigen

Correct Answer: C

The strength of a visible reaction is known as: A. prozone reaction B. absorption C. avidity D. elution

Correct Answer: C

Tissue injury in systemic rheumatic disorders such as systemic lupus erythematosus is thought to be caused by: A. cytotoxic T cells B. IgE activity C. deposition of immune complexes D. cytolytic antibodies

Correct Answer: C

To which of the following classes do the antibodies that cause hemolytic disease of the newborn belong? A. IgA B. IgE C. IgG D. IgD

Correct Answer: C

What disease is indicated by a high titer of anti-Sm (anti-Smith) antibody? A. Mixed connective tissue disease (MCTD) B. RA C. SLE D. Scleroderma

Correct Answer: C

What is measured in the CH50 assay? A. RBC quantity needed to agglutinate 50% of antibody B. Complement needed to lyse 50% of RBCs C. Complement needed to lyse 50% of antibody-sensitized RBCs D. Antibody and complement needed to sensitize 50% of RBCs

Correct Answer: C

What is the advantage of 4th-generation rapid HIV tests over earlier rapid HIV tests? A. They use recombinant antigens B. They detect multiple stains of HIV C. They detect p24 antigen D. They are quantitative

Correct Answer: C

What is the correct procedure upon receipt of a test request for human chorionic gonadotropin (hCG) on the serum form a 60-year-old man? A. Return the request; hCG is not performed on men B. Perform a qualitative hCG test to see if hCG is present C. Perform the test; hCG may be increased in testicular rumors D. Perform the test but use different standards and controls

Correct Answer: C

What is the immunologic method utilized in the flow cytometer? A. latex agglutination B. enzyme linked immunoassay C. immunofluorescence D. radioimmunoassay

Correct Answer: C

What is the interpretation when an Ouchterlony plate shows crossed lines between wells 1 and 2 (antigen is place in the center well and antisera in wells 1 and 2)? A. No reaction between wells 1 and 2 B. Partial identity between wells 1 and 2 C. Nonidentity between wells 1 and 2 D. Identity between wells 1 and 2

Correct Answer: C

What is the main difficulty associated with the development of an HIV vaccine? A. The virus has been difficult to culture; antigen extraction and concentration are extremely laborious B. Human trails cannot be performed C.Different stains of the virus are genetically diverse D. Anti-idiotype antibodies cannot be developed

Correct Answer: C

What is the most likely explanation when a patient has clinical signs of viral hepatitis but test negative for hepatitis A IgM, hepatitis B surface antigen, and hepatitis C Ab? A. Tests were performed improperly B. The patient does not have hepatitis C. The patient may be in the "core window" D. Clinical evaluation was performed improperly

Correct Answer: C

What type of antigen is used in the RPR card test? A. Live treponemal organisms B. Killed suspension of treponemal organisms C. Cardiolipin D. Tanned sheep cells

Correct Answer: C

Which T-cell malignancy may retain "helper" activity with regard to immunoglobulin synthesis by B cells? A. Hodgkin lymphoma B. acute lymphocytic leukemia (ALL) C. Sezary syndrome D. chronic lymphocytic leukemia (CLL)

Correct Answer: C

Which disease is likely to show a rim (peripheral) pattern in an immunofluorescence (IF) microscopy test for ANA? A. Mixed connective tissue disease (MCTD) B. Rheumatoid arthritis C. Systemic lupus erythematosus D. Sclerodema

Correct Answer: C

Which disease may be expected to show an IgM spike on an electrophoretic pattern? A. Hypogammaglobulinemia B. Multicystic kidney disease C. Waldenstom's macroglobulinemia D. Wiskott-Aldrich syndrome

Correct Answer: C

Which immunofluorescence pattern indicates the need for ENA testing by Ouchterlony immunodiffusion, Multiplex, or ELISA assays? A. Homogenous or solid B. Peripheral or rim C. Speckled D. Nucleolar

Correct Answer: C

Which in vitro test measures IgE levels against a specific allergen? A.Histamine release assay B. Radioimmunosorbent test (RIST) C Fluorescent allergosorbent test (FAST. D. Precipitin radioimmunosorbent test (PRIST)

Correct Answer: C

Which increase in antibody titer (dilution) best indicates an acute infection? A. From 1:2 to 1:8 B. From 1:4 to 1:16 C. From 1:16 to 1:256 D. From 1:64 to 1:128

Correct Answer: C

Which is a recognized theory of the origin of autoimmunity? A. enhanced regulatory T-cell function B. diminished helper T-cell activity C. production of antibodies that cross-react with tissue components D. deficient B-cell activation

Correct Answer: C

Which method is the most sensitive for quantitation of AFP? A. Double immunodiffusion B. Electrophoresis C. Enzyme immunoassay D. Particle agglutination

Correct Answer: C

Which method is used to test for HIV infection infants who are born to HIV-positive mothers? A. ELISA B. Western blot test C. Polymerase chain reaction D. Viral culture

Correct Answer: C

Which of the following are true statements about selective IgA deficiency: A. associated with a decreased incidence of allergic manifestation B. high concentration of secretory component in the saliva C. associated with an increased incidence of autoimmune disease D. found in approximately 1 out of every 50 persons

Correct Answer: C

Which of the following complement components is a strong chemotactic factor as well as a strong anaphylatoxin? A. C3a B. C3b C. C5a D. C4a

Correct Answer: C

Which of the following immunoglobulins is the most efficient at agglutination: A. IgG B. IgA C. IgM D. IgE

Correct Answer: C

Which of the following is a treponemal test? A. RST B. RPR C. FTA-ABS D. VDRL

Correct Answer: C

Which of the following is a true statement about Bruton agmmaglobulinemia? A. it is found only in females B. there are normal numbers of circulating B cells C. there are decreased to absent concentrations of immunoglobulins D. the disease presents with pyogenic infections 1 week after birth

Correct Answer: C

Which of the following is an organ-specific autoimmune disease? A. myasthenia gravis B. rheumatoid arthritis C. Addison disease D. progressive systemic sclerosis

Correct Answer: C

Which of the following is cleaved as a result of activation of the classical complement pathway? A. properdin factor B B. C1q C. C4 D. C3b

Correct Answer: C

Which of the following is most likely to activate the alternative pathway of complement activation? A. lipopolysaccharides B. glycoproteins C. hapten D. IgG complexed with antigen

Correct Answer: C

Which of the following is the most common humoral immune deficiency disease? A. Bruton agammaglobulinemia B. IgG deficiency C. Selective IgA deficiency D. Wiskott-Aldrich syndrome

Correct Answer: C

Which of the following is used to detect allergen specific IgE? A. RIST B. IEP C. RAST D. CRP

Correct Answer: C

Which of the following releases histamine and other mediators from basophils? A. C3a B. properdin factor B C. C1q D. C4

Correct Answer: C

Which of the following statements about immunoglobulins is true: A. immunoglobulins are produced by T lymphocytes B. IgA class is determined by the gamma heavy chain C. IgA class exists as serum and secretory molecules D. there are only 2 subclasses of IgG

Correct Answer: C

Which of the following symptoms in a young child may indicate an immunodeficiency syndrome? A. Anaphylactic reactions B. Severe raches and myalgia C. Recurrent bacterial, fungal, and viral infection D. Weigh loss, rapid heartbeat, breathlessness

Correct Answer: C

Which of the following terms describes a graft between genetically unidentical individuals belonging to the same species: A. autograft B. isograft C. allograft D. xenograft

Correct Answer: C

Which of the following test is used to detect circulating immune complexes in the serum of some patients with systemic autoimmune disease such as rheumatoid arthritis? A. Direct immunofluorescence B. Enzyme immunoassay C. Assay of cryoglobulins D. Indirect antiglobulin test

Correct Answer: C

Which outcome indicates a negative result in a complement fixation test? A. Hemagglutination B. Absence of hemagglutination C. Hemolysis D. Absence of hemolysis

Correct Answer: C

Which part of the radial immunodiffusion (RID) test system contains the antisera? A. Center well B. Outer wells C. Gel D. Antisera may be added to any well

Correct Answer: C

Which serological marker HBV (hepatitis B virus) infection indicates recovery and immunity? A. viral DNA polymerase B. HBe antigen C. anti-HBs D. HBsAg

Correct Answer: C

Which specimen is the sample of choice to evaluate latent or tertiary syphilis? A. Serum sample B. Chancre fluid C. CSF D. Joint fluid

Correct Answer: C

Which test has the greatest sensitivity for antigen detection? A. precipitin B. agglutination C. ELISA D. complement fixation

Correct Answer: C

Why is a chemiluminescent immunoassay (CIA) or enzyme immunoassay (EIA) the method of choice for detection of certain analytes, such as hormones, normally found in low concentration? A. Because of low cross reactivity B. Because of high specificity C. Because of high sensitivity D. Because test systems may be designed as both competitive and noncompetitive assays

Correct Answer: C

13. The major class of immunoglobulin found in adult human serum is A. IgA B. IgE C. IgG D. IgM

Correct Answer: C C. Immunoglobulin G is Ihe predominant class of immunoglobulin found in serum. It accounts for approximately 80% of the lolal serum immunoglobulin. The normal range is 800-1600 mg/dL.

47. The portion of an antigen that binds to an antibody or T cell receptor is called a(n) A. Allergin B. Avidin C. Epitope D. Valence

Correct Answer: C Antigens can have multiple epitopes. Each epitope can be unique, binding an antibody with a different idiotype. "Valence" refers to the number of epitopes on an antigen.

43. The antibody most frequently present in systemic lupus erythematosus is directed against A. Surface antigens of bone marrow stem cells B. Surface antigens of renal cells C. Nuclear antigen D. Myelin

Correct Answer: C Antinuclear antibody (ANA) is the most consistent feature of systemic lupus erythematosus (SLE). Although renal or nerve pathology may occur, that pathology is secondary to deposition of antigen-antibody complexes and subsequent activation of complement proteins. Bone marrow stems cells are not involved in the pathology of SLE.

40. The type of immunity that follows the injection of antibodies synthesized by another individual or animal is termed A. Artificial active B. Natural adaptive C. Artificial passive D. Natural passive

Correct Answer: C Artificial passive immunity results following the injection of antibody synthesized by another individual or animal. This type of immunity is only temporary but may be very important in providing "instant" protection from an infectious agent before the recipient would have time to actively synthesize antibody. The injected antibodies are treated as foreign proteins and are eventually cleared from the body.

3. Which of the following is characteristic of B cells? A. Phagocytic B. Participate in antibody- dependent C. Contain surface immunoglobulins D. Secrete the C5 component of complement

Correct Answer: C B cells carry surface immunoglobulins that react to a specific antigen. The antigen can then be internalized processed and presented to an appropriate T helper cell. B cells are not phagocytic, nor do they participate in antibody-dependant cellular cytotoxicity (ADCC) reactions. Complement proteins are secreted by hepatocytes.

62. Interaction between B and T helper cells involves A. MHC II molecule on B cell binding to MHC I molecule on the T cell B. MHC II molecule on B cell binding to CD3 on the T cell C. Foreign antigen on B cell binding to T cell receptor D. CD3 molecule on B cell binding to T cell receptor

Correct Answer: C B cells have the ability to present antigen (immunogen) to T helper cells. This interaction involves several surface molecules. The antigen is complexed with MHC II on the surface of the B cell. CD4 on the T cell interacts with MHC II, whereas the T cell receptor binds the antigen.

23. In bone marrow transplantation, immunocompetent cells in the donor marrow may recognize antigens in the recipient and respond to those antigens. This phenomenon is an example of: A. Acute rejection B. Chronic rejection C. Graft versus host disease D. Hyperacute rejection

Correct Answer: C Bone marrow transplants by their nature contain immunologically competent cells: B cells and T cells in particular. Unless the transplanted marrow is HLA-matched perfectly to the donor, the immunocompetent cells in the transplant will recognize and react against the nonself HLAs of the recipient's tissues. This phenomenon is known as graft-versus-host disease, because the graft attempts to reject its host. Acute rejection, chronic rejection, and hyperacute rejection are examples of mechanisms a recipient's immune system uses to reject a graft.

10. A young woman show increased susceptibility to pyogenic infections. Upon assay, she show a low level of C3. Which of the following statements is probable true? A. She has an autoimmune disease with continual antigen-antibody activity causing consumption of C3. B. She has DiGeorge syndrome. C. She has decreased production of C3. D. She may produce an inactive form of C2, a precursor of C3.

Correct Answer: C C3 may be decreased due to a genetic defecl lhat causes deficient production. In certain autoimmune disorders, such as systemic lupus erylhematosus, continual complemenl activation leads lo low levels; however, susceptibility to pyogenic infections is not a feature of autoimmune diseases. DiGeorge syndrome is a deficiency in T cells, and complement protein C2 is not a precursor of C3.

65. Which complement protein is present in the greatest concentration in human serum? A. C1 B. C2 C. C3 D. C4

Correct Answer: C Complement protein C3 has a serum concentration of about 1300 (xg/mL, which makes it the complement protein present in the greatest concentration. The second highest concentration of complement protein is C4 (600 |xg/mL). C3 is cleaved into fragments: C3a and C3b.

88. Cells that can act as antigen-presenting cells for exogenous antigens include A. All nucleated cells B. Endothelial cells C. B lymphocytes D. T lymphocytes

Correct Answer: C Exogenous antigens are nonself antigens derived from infectious agents or immunizing preparations. Exogenous antigens are processed for presentation to specific T cells by specialized cells collectively referred to as antigen-presenting cells (APCs). APCs for exogenous antigens include B cells, macrophages, monocytes, and dendritic cells.

29. A kidney transplant from one identical twin to another is an example of a(n) A. Allograft B. Autograft C. Isograft D. Xenograft

Correct Answer: C Identical twins have the same genetic makeup. Grafts between them would be isografts or syngeneic grafts. Autografts are transplantations from one site to another in the same individual. Xenograft refers to transplantation between different species. Transplantationbetween two nonidentical individuals of the same species is called an allograft.

59. Antibodies that bind to the same epitope are of the same A. Allotype B. Autotype C. Idiotype D. Isotype

Correct Answer: C Idiotype of an antibody refers to the antigen specificity of the molecule. The isotype is the different classes and subclasses of antibodies (e.g., IgG, IgM, etc.). "Allotype" refers to different alle- les of the same isotype. Genetically different individuals will produce antibodies of the same isotype, but they would have a different allotype

81. A patient report states presence of serum antibodies to OspC. What disease does the patient most likely have? A. Syphilis B. Strep throat C. Lyme disease D. Rubella

Correct Answer: C IgM antibody to OspC is an important early marker in the diagnosis of Lyme disease. This antibody, along with several others, is often detected by Western blot. Antibodies to p35, p39, and the flagellin subunits p37 and p41 are also useful in diagnosing this disease.

12. An important part of the nonspecific immune response is (are): A. B cells B. Basophils C. Complement cascade D. Cytotoxic T lymphocytes

Correct Answer: C Important parts of an animal's nonspecific immune response include phagocytosis, inflammation, and complement activation. In a nonspecific immune response, the animal responds in much Ihe same way to all invaders. B cells and cytoloxic T lymphocytes responded to specific antigens and are, therefore, involved in the specific immune response. Basophils are involved in type I hypersensitivily reactions.

19. When performing the enzyme-multiplied immunoassay technique (EMIT), how is the liagand in the patient's serum detected? A. Agglutinates by binding to antibody-coated latex beads B. Binds to enzyme-labeled antibody C. Competes with enzyme-labeled antigen for binding to a specific antibody D. Forms antibody-antigen complex and precipitates

Correct Answer: C In the EMIT, a ligand (antigen) in a sample competes with an enzyme-labeled ligand for binding to a specific antibody. The labeled ligand is designed so that following antibody binding, the enzyme is inactive. As the ligand concentration in the test sample increases, more enzyme-labeled ligand remains unbound, resulting in greater enzyme activity.

17. Which class of immunoglobulin binds to basophils and mast cells to mediate immediate hypersensitivity reactions? A. IgA B. IgD C. IgE D. IgG

Correct Answer: C Mast cells and basophils have surface recep- tors (FceRI) for the Fc portion of IgE. When IgE molecules, attached to the surface of mast cells and basophils, bind the allergen they are specific for, this triggers the cells to degranulate, producing the symptoms of immediate type I hypersensitivity. The main function of IgE appears to be the ability to trigger an immune response, thereby recruiting plasma factors and effector cells to areas of trauma or parasite infection.

20. Severe combined immunodeficiency (SCID) is an: A. Immunodeficiency with decreased B cells and neutrophils B. Immunodeficiency with lymphocytopenia and eosinophilia C. Immunodeficiency with decreased or dysfunctional T and B cells D. Immunodeficiency with decreased lymphocytes and decreased complement concentration

Correct Answer: C SCID is defined as a condition in which adaptive immune responses (i.e., cell-mediated and humoral-mediated immune responses) do not occur because of a lack of T and B cell activity. A number of genetic defects can lead to this condition. Children born with SCID need to live in a sterile environment, and they have a short life expectancy.

34. Diagnostic reagents useful for detecting antigen by the coagglutination reaction may be prepared by binding antibody to killed staphylococcal cells via the Fc receptor of staphylococcal protein A. The class of antibody bound by this protein is A. IgA B. IgD C. IgG D. IgM

Correct Answer: C Staphylococcal protein A binds only the IgG class (subclasses IgGl, IgG2, and IgG4) of immunoglobulin. Binding occurs via the Fc portion of the antibody molecule, leaving the Fab portion available to bind antigen in an immunologic assay. Binding of the Fab portion to test antigen causes agglutination of the staphylococcal cells (coagglutination).

74. A cut on a person's finger becomes contaminated with the bacterium Staphylococcus aureus. The first response by the immune system consists of activity of A. B cells B. Monocytes C. Neutrophils D. T cells

Correct Answer: C The first response to invading bacteria is mounted by the innate immune system. The innate immune system, although it lacks the specificity of the adaptive immune system, is nonetheless effective at handling many invading bacteria. The first response by the innate immune system consists of an influx of neutrophils into the tissue invaded by bacteria. Monocytes and macrophages, although they are phagocytic cells and part of the innate immune system, play only a minor role in the initial response to bacterial invasion.

A hospital employee received the final dose of hepatitis B vaccine 3 weeks ago. She wants to donate blood. Which of the following results are expected form the hepatitis screen, and will she be allowed to donate blood? A. HBsAg, positive; anti-HBc, negative-she may donate B. HBsAg, negative; anti-HBc, positive-she may not donate C. HBsAg, positive; anti-HBc, positive-she may not donate D. HBsAg, negative; anti-HBc, negative-she may donate

Correct Answer: D

A laboratory is evaluating an enzyme-linked immunosorbent assay (ELISA) for detecting an antibody to cyclic citrullinated peptide (CCP), which is a marker for rheumatoid arthritis. The laboratory includes serum from healthy volunteers and patients with other connective tissue disease in the evaluation. These specimens determine which factor of the assay? A. Sensitivity B. Precision C. Bias D. Specificity

Correct Answer: D

A monoclonal spike of IgG, Bence Jones proteinuria, and bone pain are usually associated with: A. Burkitt lymphoma B. Bruton disease C. severe combined immunodeficiency disease D. multiple myeloma

Correct Answer: D

A patient with symptoms associated with SLE and scleroderma was evaluated by immunofluorescence microscopy for ANAs using the HEp-2 cell lines as substrate. The cell line displayed a mixed pattern of fluorescence that could not be separated by serial dilutions of the serum. Which procedure would be most helpful in determining the antibody profile of this patient? A. Use of a different tissue substrate B. Absorption of the serum using the appropriate tissue extract C. Ouchterlony technique D. ELISA tests for specific antibodies

Correct Answer: D

A patient's serum IgA as measured by radial immunodiffusion (RID) was 40 mg/dL. Another laboratory reported IgA absent. A possible explanation for this discrepancy is that the: A. rabbit antiserum was used in the RID plates and rabbit antisera should not be utilized in RID assays B. IgA has an Fc deletion that cause complex formation in vivo C. IgA antiserum has kappa specificity D. patient serum has antibodies against a protein in the antiserum in the agarose of the RID utilized by the first lab

Correct Answer: D

A patient's specimen is strongly positive in an ANA ELISA. Which of the following would not be an appropriate follow up to this result? A. Immunofluorescence test on HEp-2 cells B. Specific ENA ELISA tests C. Specific anti-DNA ELISA D. Rheumatoid factor assay

Correct Answer: D

A renal transplant recipient is found to have a rising creatinine level and reduced urine output. The physician order a "Urine PCR" assay. When you call to find out what organism the physician wants to identify, you are told: A. Hepatits C virus B. Legionella pneumophila C. EBV D. BK virus

Correct Answer: D

A series of 8 tubes are set-up with 0.79 mL of diluent in each. A serial dilution is performed by adding 10 uL of serum to the first tube and then transferring 10 uL through each remaining tube. What is the serum dilution of tube 7? A. 1:2.431 x 10^13 B. 1:2.621 x 10^13 C. 1:1.920 x 10^13 D. 1:2.097 x 10^13

Correct Answer: D

A single, reliable screening test for detecting neonatal infection in the absence of clinical signs is: A. serum immunoelectrophoresis B. differential leukocyte count C. CD4 cell counts D. quantitative serum IgM determination

Correct Answer: D

A transplant patient began to show signs of rejection 8 days after receipt of the transplanted organ, and the organ was removed. What immune elements might be found in the rejected organ? A. Antibody and complement B. Primarily antibody C. Macrophages D. T cells

Correct Answer: D

After a penicillin injection, a patient rapidly develops respiratory distress, vomiting and hives. This reaction is primarily mediated by: A. IgG B. IgA C. IgM D. IgE

Correct Answer: D

All of the following test may be abnormal in a type III immune complex reaction except: A. C1q-binding assay by ELISA B. Raji cell assay C. CH50 level D. Mitogen response

Correct Answer: D

All of the following tests measure phagocyte function except: A. Leukocyte adhesion molecule analysis B. Di Hydro rhodamine reduction assay C. NBT test D. IL-2 (interleukin-2) assay

Correct Answer: D

All tubes (dilutions) except the negative control are positive for cold agglutinins. This indicates: A. Contaminated red cells B. A rare antibody against red cell antigens C. The sample was stored at 4C prior to separating serum and cells D. Further serial dilution is necessary

Correct Answer: D

An IFE revealed excessive amounts of polyclonal IgM and low concentrations of IgG and IgA. What is the most likely explanation of these findings and the best course of action? A. Proper amounts of antisera were not added; repeat both tests B. Test specimen was not added properly; repeat both procedures C. Patient has a common variable immunodeficiency: perform B-cell count D. Patient has immunodeficiency with hyper-M: perform immunoglobulin levels

Correct Answer: D

An example of a live attenuated vaccine used for human immunization is: A. rabies B. tetanus C. hepatitis B D. measles

Correct Answer: D

Anti-RNA antibodies are often present in individuals having an anti-nuclear antibody immunofluorescent patterns that is: A. speckled B. rim C. diffuse D. nucleolar

Correct Answer: D

Anti-glomerular basement membrane antibody is most often associated with this condition: A. systemic lupus erythematosus B. celiac disease C. chronic active hepatitis D. Goodpasture disease

Correct Answer: D

Antibodies are produced by: A. killer cells B. marrow stem cells C. mast cells D. B cells

Correct Answer: D

Antibody allotype is determined by the: A. constant region of heavy chain B. constant region of light chain C. variable regions of heavy and light chains D. constant regions of heavy and light chains

Correct Answer: D

Antihistamines like Benadryl: A. depress IgE production B. block antigen binding to surface IgE C. bind histamine D. block H1 histamine receptors

Correct Answer: D

Avidity may be defined as the: A. degree of hemolysis B. titer of an antigen C. dilution of an antibody D. strength of a reacting antibody

Correct Answer: D

Blood is drawn from a patient for serological tests for a viral disease at the time of onset and again 4 weeks later. The results of the tests are considered diagnostic if the: A. first antibody titer is 2x the second B. first and second antibody titers are equal C. first antibody is 4x the second D. second antibody titer is at least 4x the first

Correct Answer: D

Cells from a patient with hairy cell leukemia have immunologic and functional features of: A. mast cells and B lymphocytes B. B lymphocytes and T lymphocyte C. granulocytes and monocytes D. B lymphocytes and monocytes

Correct Answer: D

Cholesterol is added to the antigen used in flocculation test for syphilis to: A. destroy tissue impurities present in the alcoholic beef heart extract B. sensitize the sheep RBCs C. decrease specificity of the antigen D. increase sensitivity of the antigen

Correct Answer: D

Excess antigen in precipitation gel reactions will: A. have no effect on the precipitate reaction B. not dissolve precipitate after formation C. enhance the precipitate reaction D. dissolve the precipitate after formation

Correct Answer: D

False-positive rheumatoid factor in agglutination and nephelometric methods can be due to elevated levels of: A. cryoglobulin B. histidine-rich-glycoprotein C. aspartame D. C1q

Correct Answer: D

Hereditary deficiency of late complement components (C5, C6, C7 or C8) can be associate with which of the following conditions? A. pneumoccal septicemia B. small bowel obstruction C. systemic lupus erythematosus D. a systemic gonococcal infection if exposed

Correct Answer: D

High titers of antimicrosomal antibodies are most often found in: A. rheumatoid arthritis B. systemic lupus erythematosus C. chronic hepatitis D. thyroid disease

Correct Answer: D

If only anti-HBS is positive, which of the following can be ruled out? A. Hepatits B virus vaccination B. Distant past infection with hepatitis B virus C. Hepatitis B immune globulin (HBIG) injection D. Chronic hepatits B virus infection

Correct Answer: D

IgM antibodies are frequently hemolytic because of: A. their dimeric structures B. the molecule's 5 antigen binding sites C. their sedimentation coefficient of 7-15 S D. their efficient ability to fix complement

Correct Answer: D

Immunodeficiency with thrombocytopenia and eczema is often referred to as: A. DiGeorge syndrome B. Bruton agammaglobulinemia C. ataxia telangiectasia D. Wiskott-Aldrich syndrome

Correct Answer: D

In a positive anti-streptolysin "O" enzyme inhibition test, the patient's: A. streptolysin "O" enzyme in the patient serum neutralizes the anti-streptolysin "O" reagent, resulting in no hemolysis B. red blood cells are hemolyzed by the streptolysin "O" enzyme in the reagent C. anti-streptolysin "O" neutralizes the streptolysin "O" reagent, resulting in hemolysis D. anti-streptolysin "O" inhibits the reagent streptolysin "O"s, resulting in no hemolysis

Correct Answer: D

In laser flow cytometry, applying a voltage potential to sample droplets as they stream past the light beam and using charged deflector plates results in: A. an emission of red fluorescence from cells labeled with fluorescein isothiocyanate B. an emission of green fluorescence form cells labeled with rhodamine C. a 90% light scatter related to cell size D. the separation of cells into subpopulations based on their charge

Correct Answer: D

In pernicious anemia, which of the following antibodies is characteristically detected? A. antimitochondrial B. anti-smooth muscle C. anti-DNA D. anti-parietal cell

Correct Answer: D

In testing for DiGeorge's syndrome, what type of laboratory analysis would be most helpful in determining the number of mature T cells? A. Complete blood count B. Nitroblue tetrazolium (NBT) test C. T-cell enzyme assays D. Flow cytometry

Correct Answer: D

In the FTA-ABS test, the presence of a beaded pattern of fluorescence along the treponeme indicates: A. positive identification of Treponema pallidum B. presumptive diagnosis of active syphilis C. presence of nontreponemal antibody (NTA) D. false-positive reaction

Correct Answer: D

In the indirect fluorescent anti-nuclear antibody test, a homogenous pattern indicates the presence of antibody to: A. RNP B. Sm C. RNA D. DNA

Correct Answer: D

Incompatibility by which of the following procedures is an absolute contraindication to allotrasplantation? A. MLC (mixed lymphocyte culture) B. HLA typing C. Rh typing D. ABO grouping

Correct Answer: D

Initiation of the activation mechanism of the alternative complement pathway differs from that of the classical pathway in that: A. antigen-antibody complexes containing IgM or IgG are required B. endotoxin alone cannot initiate activation C. C1 component of complement is involved D. antigen-antibody complexes containing IgA or IgE may initiate activation

Correct Answer: D

Interpret the following results for HIV infection: -HIV1,2 ELISA: positive -HIV-1 Western blot: Indeterminate HIV-1 p24 antigen: negative A. Positive for antibodies to human immunodeficiency virus, HIV-1 B. Positive for antibodies to human immunodeficiency virus, HIV-2 C. Cross reaction; biological false-positive result D. Additional testing required

Correct Answer: D

Which of the following has been associate with patients who have homozygous C3 deficiency? A. undetectable hemolytic complement activity in the serum B. systemic lupus erythematosus C. no detectable disease D. a lifelong history of life-threatening infections

Correct Answer: D

Which of the following is the "membrane attack complex" of complement activation? A. C1 B. C3 C. C4, C2, C3 D. C5b, C6, C7, C8, C9

Correct Answer: D

Which of the following is the most sensitive test to detect congenital syphilis? A. VDRL B. RPR C. Microhemagglutinin test for T. pallidum (MHA-TP) D. Polymerase chain reaction (PCR)

Correct Answer: D

Which of the following mediator is released during T-cell activation A. immunoglobulins B. thymosin C. serotonin D. cytokines

Correct Answer: D

Which of the following methods is least likely to give a definitive result for the diagnosis of RA? A. Nephelometric measurement fo anti-IgG B. Agglutination testing for rheumatoid factor C. Anti CCP D. Immunofluorescence testing for antinuclear antibodies

Correct Answer: D

Which of the following serial dilutions contains an incorrect factor? A. 1:4, 1:8, 1:16 B. 1:1, 1:2, 1:4 C. 1:5, 1:15, 1:45 D. 1:2, 1:6, 1:12

Correct Answer: D

Which of the following statements regarding infection with hepatitis D virus is true? A. Occurs in patients with HIV infection B. Does not progress to chronic hepatitis C. Occurs in patients with hepatitis B D. Is not spread through blood or sexual contact

Correct Answer: D

Which statement best describes passive agglutination reactions used for serodiagnosis? A. Such agglutination reactions are more rapid because they are a single-step process B. Reactions require the addition of a second antibody C. Passive agglutination reactions require biphasic incubation D. Carrier particles for antigen such as latex particles are used

Correct Answer: D

You are evaluating an ELISA assay as a replacement for your immunofluorescent antibulcear antibody tes. You test 50 specimens in duplicate on each assay. The ELISA assay uses a HEp-2 extract as its antigen source. The correlation between the ELISA and the IFA test is only 60% (30 of 50 specimens agree). Which of the following is the next best course of action? A. Test another 50 specimens B. Perform a competency check on the technologists who performed the tests C. Order a new lot of both kits and the retest on the new lots D. Refer the discrepant specimens for testing by another method

Correct Answer: D

Your cytology laboratory refers a Papanicolaou smear specimen to you for an assay designed to detect the presence of a virus associated with cervical cancer. You perform: A. An ELISA assay for anti-HSV-2 antibodies B. A molecular assay HSV-2 C. An ELISA assay for HPV antibodies D. A molecular assay for HPV

Correct Answer: D

72. In the classical pathway of complement activation, A. C3 is activated by binding C-reactive protein B. The sequence of activation is C1, C2, C3, C4 C. C1q is activated by the presence of a single Fab region D. Activation by antibody requires one IgM or two IgG molecules

Correct Answer: D D. Complement attaches to the Fc portion of the antibody molecule. At least two Fc binding sites are required for C l q to attach. Therefore, activation requires two IgG molecules or a single molecule of IgM, which is a pentamer. The C proteins were named in order of discovery. The correct reaction sequence is Cl, C4, C2, C3. As the last step of this reaction sequence, C3 is split into C3a and C3b

58. Monoclonal antibodies are produced by A. Cultured T cells B. Human plasma cells C. Mouse plasma cells D. Hybridomas

Correct Answer: D A monoclonal antibody is produced by a single cell or clone. Plasma cells obtained from an immunized animal and subsequently fused with myeloma cells result in a hybrid myeloma or hybridoma that will indefinitely secrete a specific antibody. Hybridomas have been prepared from mouse and human plasma cells fused with myeloma cells. T cells do not produce antibodies.

11. What is the predominant type of antibody found in the serum of neonates born after full-term gestation? A. Infant IgA B. Infant IgG C. Infant IgM D. Maternal IgG

Correct Answer: D Antibody production is immunogen induced. Because the fetus develops in a sequestered site, it makes very little immunoglobulin. Maternal IgG crosses the placenta and is the primary antibody found in infant's circulation.

30. In Bruton disease, measurement of serum immunoglobulins would show: A. Elevated levels of IgE B. Elevated levels of IgG C. Normal levels of IgG and IgM but reduced levels of IgA D. The absence of all immunoglobulins

Correct Answer: D Bruton disease is a congenital form of agam- maglobulinemia. It is a sex-linked phenomenon that affects males. Because B cells are not produced, affected males have levels of IgA, IgD, IgE, and IgM undetectable by routine assays. IgG may be absent or present at very low levels.

70. Which of the following is characteristic of contact hypersensitivity reactions? A. Caused by preformed IgE antibody B. Characterized by infiltration of neutrophils into the area of reaction C. The primary symptoms often occur in the respiratory tract D. Usually due to a hapten

Correct Answer: D Contact dermatitis is a cell-mediated hypersensitivity reaction. The offending substance is typically a hapten that combines with a carrier molecule on the skin surface. The hapten-carrier complex is recognized by T cells. IgE mediates immediate hypersensitivity reactions such as hay fever and some forms of asthma.

28. Corneal tissue may be transplanted successfully from one patient to another because: A. The cornea is nonantigenic B. Corneal antigens do not activate T cells C. Anti-corneal antibodies are easily suppressed D. The cornea occupies a privileged site not usually seen by the immune system

Correct Answer: D Corneas are readily transplanted from one individual to another. This is because the cornea is nonvascularized and is a sequestered site. Thus the immune system of the host does not "see" the cornea and recognize it as foreign

66. An autoimmune disease causing destruction of pancreatic cells can result in: A. Hashimoto disease B. Multiple sclerosis C. Myasthenia gravis D. Type 1 diabetes

Correct Answer: D Destruction of the beta cells in the pancreas results in type 1 diabetes. An autoimmune response destroys the insulin-producing cells. The immune response is probably due to molecular mimicry. Cytotoxic T cells and antibodies directed against an infectious agent cross react to the beta cells.

86. The first serologic marker to appear in patients with acute hepatitis B virus infection is A. Anti-HB B. Anti-HBC C. Anti-HBe D. HBsAg

Correct Answer: D HBsAg is the first serologic marker occurring in patients with hepatitis B virus infection. The antigen appears about 3-5 weeks before symptoms appear. About 2—4 weeks later, anti-HBc, primarily of the IgM class, begins to appear.

90. Why does vaccination against hepatitis B virus (HBV) also prevent hepatitis D virus (HDV) infections? A. An immunogen form HBV in the vaccine is also associated with HDV B. The HBV vaccine induces formation of heterophile antibodies that cross react with HDV. C. The HBV vaccine stimulates liver cells to produce antiviral molecules active against all hepatitis viruses D. HDV requires the host to be concurrently infected with HBV

Correct Answer: D HDV requires HBsAg produced by HBV-infected cells. HDV, therefore, requires the host to be concurrently infected with HBV. The HBV vaccine prevents HBV infection and also HDV infection.

2. A hapten is A. Half of an immunoglobulin molecule B. A carrier molecule for an antigen that is not antigenic alone C. An immunoglobulin functional only in the presence of complement D. A determinant capable of stimulating an immune response only when bound to a carrier

Correct Answer: D Haptens are substances that are not immunogenic by themselves. These molecules are not large or complex enough to stimulate the immune system. When bound to a carrier, they are capable of stimulating a specific immune response.

83. Hashimoto disease is an autoimmune disease primally involving the A. Kidneys B. Liver C. Lungs D. Thyroid gland

Correct Answer: D Hashimoto disease is a type of thyroiditis due to an autoimmune disease. Patients produce autoantibodies and T cells that respond to thyroid antigens. This results in inflammation and swelling of the thyroid gland (goiter). The autoantibody blocks the uptake of iodine, which results in a decrease in the production of thyroid hormones (hypothyroidism).

25. Which of the following describes a direct immunofluorescence assay? A. Conjugated reagent antigen reacts with antibodies to form antigen-antibody complexes B. Antigens react with unlabeled antibody forming antigen-antibody complexes that attach to labeled antibodies C. A dye is attached to a molecule and it reacts with an immune complex to produce a color D. Conjugated reagent antibody reacts with antigen to form antigen-antibody complexes

Correct Answer: D In a direct imrnunofluorescence assay, a fluorescent molecule is linked to an antibody. This complex is often called a conjugate. Clinical material is fixed onto a microscope slide, and the conjugate is added. After a wash step, the slide is examined with a microscope using UV light. If antigen specific to antibody was present in the clinical specimen, fluorescence will be seen.

41. Innate immunity includes A. Anamnestic responce B. Antibody production C. Cytotoxic T cell activity D. Phagocytosis by polymorphonuclear cells

Correct Answer: D Innate, or nonspecific, immunity refers to host defenses that are in general present at birth and do not require immunogen stimulation. Phagocytosis of bacteria by polymorphonuclear cells is an example. Cytotoxic T cell activity is part of the adaptive cell-mediated immune response, and antibody production is the mechanism of protection in the adaptive humoral-mediated immune response.

48. Identical antibodies produced from a single clone of plasma cells describes A. Reagin B. Cold agglutinins C. Heterophile antibodies D. Monoclonal antibodies

Correct Answer: D Monoclonal antibodies are derived from a single clone of plasma cells. Plasma cells are fused with a cancerous myeloma cell. Reagin has two meanings: it can refer to the antibody produced during syphilis or it can refer to IgE. Cold agglutinins are antibodies that agglutinate in cold temperatures (e.g., 4°C). Heterophile antibodies are antibodies produced following exposure to an immunogen that are able to bind a similar but different molecule.

46. One cause of a false-positive (VDRL) test is A. Brucellosis B. Treponema pallidum infection C. Rocky Mountain spotted fever D. Systemic lupus erythematosus

Correct Answer: D Patients with connective tissue disorders such as systemic lupus erythematosus may show a false positive reaction in the VDRL test. Other causes of false positives include rheumatic fever, infectious mononucleosis, malaria, and pregnancy. Treponema pallidum subsp. pallidum is the causative agent of syphilis.

76. A soluble antigen and soluble antibody reacting to form an insoluble product describes A. Agglutination reactions B. Heterophile reactions C. Labeled reactions D. Precipitation reactions

Correct Answer: D Precipitation reactions involve both soluble antigens and antibodies. These reactions are typically detected in agarose gels. With agglutination reactions, one of the reactants is soluble and the other is insoluble. A reactant is made insoluble by combining with a carrier particle such as latex beads.

22. The serologically detectable antibody preoduced in rehamatoid arthritis (RA) is primarily of the class A. IgA B. IgE C. IgG D. IgM

Correct Answer: D Rheumatoid factor (RF) is an immunoglobulin that reacts with antigenic determinants on an IgG molecule. Although they may be of several types, the one that is easily serologically detectable is IgM. This is because of the agglutination activity of the molecule. RF tests are commonly used in the diagnosis of rheumatoid arthritis.

55. A human cell with CD8 on its surface is most likely a A. B cell B. Monocyte C. T helper cell D. Cytotoxic T cell

Correct Answer: D The CDS molecule is found primarily on cytotoxic T lymphocytes. T helper cells possess CD4 on their surface, as do several other cell types. CD3 is a marker found on most T cells.

82. Patient serum is mixed with a suspension of guinea pig antigen. When the sample is then mixed with horse red blood cells, agglutination occurs. This is suggestive of an infection caused by A. Borrelia burgdorferi B. Hepatitis B virus C. Hepatitis C virus D. Epstein-Barr virus

Correct Answer: D The Davidsohn differential test can be used to detect heterophile antibodies produced during infectious mononucleosis. These antibodies are not adsorbed by guinea pig antigens. Therefore, the antibodies are available to agglutinate horse red blood cells. Forssman heterophile antibodies are absorbed by guinea pig antigens and would not agglutinate the horse red blood cells.

51. The Fab portion of an antibody A. Binds T cell receptor B. Consists of two light chains only C. Consists of two heavy chains only D. Contains the hypervariable region

Correct Answer: D The Fab portion of an antibody contains the hypervariable region. This portion of the molecule has a variable sequence of amino acids that affects the three-dimensional structure of the molecule and, therefore, determines the specificity (idiotype) of the antibody. This region contains the amino terminal portion of the two light chains and the two heavy chains.

57. Which of the following statements applies to the Fc fragment of an immunoglobulin molecule? A. It consists of the entire heavy chain B. It contains the variable region of the heavy chain C. It contains the antigen binding sites of the molecules D. It is the region of the molecule that binds to receptors on various white blood cells

Correct Answer: D The Fc (crystalline) fragment of an immunoglobulin is produced by papain digestion of an immunoglobulin monomer. The Fc portion of antibodies binds to specific Fc receptors on the surface of some white blood cells. Only part of the heavy chain is found in the Fc fragment. The Fab fragment contains the antigen-combining sites of both the heavy chains and the light chains.

15. Which class of immunoglobulin possesses 10 antigenic binding sites? A. IgA B. IgD C. IgG D. IgM

Correct Answer: D The IgM molecule is a penlamer that con- tains 10 binding sites. However, the actual valence falls to 5 with larger antigen molecules, probably because of steric restrictions. IgA, IgG, IgD, and IgE monomers each have two antigenic binding sites.

98. A male infant had been well until about 5 months of age, at which time he was diagnosed as having otitis media and bronchitis caused by Haemophilus influenzae. Over the next several months he presented with streptococcal pneumonia several times. At 10 months of age a serum protein electrophoresis showed a virtual lack of gamma globulins. Quantitative serum levels were as follows: 75 mg/dL IgG and undetectable levels of IgM, IgA, and IgE. There were a normal number of T cells, and they exhibited normal mitogen stimulation. What disease does this child most likely suffer from? A. Combined immunodeficiency B. DiGeorge syndrome C. Iatrogenic immunodeficiency D. X-linked agammaglobulinemia

Correct Answer: D The case history is typical of a child with X-linked agammaglobulinemia. He presented with chronic and recurrent infections beginning at 5 months of age, when transplacentally acquired IgG had declined. Normal IgG serum level is about 800-1200 mg/dL. The infant had normal T cell function, which rules out combined immunodeficiency and DiGeorge syndrome. "latrogenic immunodeficiency" refers to an immunodeficiency following therapy prescribed by a physician.

27. After exposure to antigen, the first antibodies that can be detected belong to the class A. IgA B. IgE C. IgG D. IgM

Correct Answer: D The first B cells to respond to antigen differentiate into plasma cells that produce IgM antibody. Later in the immune response, stimulated B cells undergo a phenomenon called "class switching" and begin to produce antibodies of the IgG, IgA, and IgE classes. High concentration of IgM in patient serum is indicative of a recent infection.

77. Which of the following is an example of a treponemal antigen test used for the diagnosis of syphilis? A. CRP B. RPR C. VDRL D. FTA-ABS

Correct Answer: D The fluorescent treponema antibody absorbance (FTA-ABS) test is often used as a confirmatory test for syphilis. Treponema pallidum subsp. pallidum, the causative agent of syphilis, is the source of the antigen. The rapid plasma reagin (RPR) and Venereal Disease Research Laboratory (VDRL) are diagnostic tests for syphilis that use nontreponemal antigen. C-reactive protein (CRP) is not involved in syphilis testing.

69. Which of the following complement proteins is part of the membrane attack complex? A. C1 B. C3 C. C4 D. C5

Correct Answer: D The membrane attack complex forms following the binding of C5 to a biologic membrane. The complex is formed by the sequential addition of C6, C7, C8, and C9. When C5-C8 complex with C9, a tubule is formed that bridges the cell membrane.

67. An Ouchterlony gel diffusion plate is depicted in Color Plate 24. The center well contains antibody, and the peripheral wells contain antigens labeled 1 through 4. What is the relationship between the antigens in wells 2 and 3? A. 2 is part of 3 B. 3 is part of 2 C. They are identical D. They are unrelated

Correct Answer: D The two antigens are not related. There are two different antibodies that are able to react with the two antigens, forming precipitin lines that cross. If the antigens were identical, a smooth curve precipitation line would have formed.

18. Type I hypersensitivity is: A. Associated with complement-mediated cell lysis B. Due to immune complex deposition C. Mediated by activated macrophages D. An immediate allergic reaction

Correct Answer: D Type I hypersensitivity reactions occur immediately after second exposure to an allergen. On the first, or primary, exposure, IgE specific to the allergen is produced. The IgE binds to Fc receptors on the surface of basophils and mast cells. Immune complexes and complement are not involved in the response.

The curve below was obtained by adding increasing amounts of a soluble antigen to fixed volumes of monospecific antiserum: See BOC pg 238 Pic 1 The area in which the addition of more antibody would result in the formation of additional precipitate is: A. A B. B C. C D. D

Correct answer: B

A patient who is blood group O is accidentally transfused with group A blood and develops a reaction during the transfusion. What antibody is involved in this type II reaction? A. IgM B. IgE C. IgG and IgE D. IgG

Correct Answer: A

A patient with joint swelling and pain tested negative for serum RF by both latex agglutination and ELISA methods. What other test would help establish a diagnosis of RA in this patient? A. Anti CCP B. ANA testing C. Flow cytometry D. Complement levels

Correct Answer: A

A false-negative cold agglutinin test may result if: A. the specimen is centrifuged at room temperature B. the cold agglutinin demonstrates anti-I specificity C. the specimen is refrigerated prior to serum separation D. adult human O red cells are used in the assay

Correct Answer: C

A marked decrease in the CD4 lymphocytes and decrease in the CD4/CD8 ration: A. in diagnostic for bacterial septicemia B. may be seen inmost hereditary immunodeficiency disorders C. is associated with a viral induced immunodeficiency D. is only seen in patients with advanced disseminated cancer

Correct Answer: C

Which hepatitis antibody confers immunity against reinfection with hepatitis B virus? A. Anti-HBc IgM B. Anti-HBc IgG C. Anti-HBe D. Anti-HBs

Correct Answer: D

Which is most likely a positive Western blot result for infection with HIV? A. Band at p24 B. Band at pg60 C. Bands at p24 and p31 D. Bands at p24 and gp 120

Correct Answer: D

A patient was tested for syphilis by the RPR method and was reactive. An FTA-ABS test was performed and the result was negative. Subsequent testing showed the patient to have a high titer of method. Which routine laboratory test is most likely to be abnormal for this patient? A. Activated partial thromboplastin time (APTT) B. Antismooth muscle antibodies C. Aspartate aminotransferase (AST) D. C3 assay by immunonephelometry

Correct Answer: A

Which of the following is decreased in serum during the active stages of systemic lupus erythematosus? A. anti-nuclear antibody B. immune complexes C. complement (C3) D. anti-DNA

Correct Answer C

10. All of the following are immunologic functions of complement except: A. Induction of an antiviral state B. Opsonization C. Chemotaxis D. Anaphylatoxin formation 3.1 - Basic Principles of Immunology

Correct Answer: A

13. How is complement activity destroyed in vitro? A. Heating serum at 56C for 30 min B. Keeping serum at room temperature of 22 C for 1 hour C. Heating serum at 37C for 45 min D. Freezing serum at 0C for 24 hours 3.1 - Basic Principles of Immunology

Correct Answer: A

15. Which region of the immunoglobulin molecule can bind antigen? A. Fab B. Fc C. Cl D. Ch 3.1 - Basic Principles of Immunology

Correct Answer: A

19. Which immunoglobulin appears in highest titer in the secondary response? A. IgG B. IgM C. IgA D. IgE 3.1 - Basic Principles of Immunology

Correct Answer: A

20. Which immunoglobulin can cross the placenta? A. IgG B. IgM C. IgA D. IgE 3.1 - Basic Principles of Immunology

Correct Answer: A

29. Macrophages produce which of the following proteins during antigen processing? A. IL-1 and IL-6 B. y-interferon C. IL-4, IL-5, IL-10 D. Complement components C1 and C3 3.1 - Basic Principles of Immunology

Correct Answer: A

3. What type of B cells are formed after antigen stimulation? A. Plasma cells and memory B cells B. Mature B cells C. Antigen-dependent B cells D. Receptor-activated B cells 3.1 - Basic Principles of Immunology

Correct Answer: A

A 16-year-old boy with infectious mononucleosis has a cold agglutinin titer of 1: 2000. An important consideration of this antibody's clinical relevance is the: A. thermal range B. titer 4C C. specificity D. light chain type

Correct Answer: A

A DPT vaccination is an example of: A. active humoral-mediated immunity B. passive humoral-mediated immunity C. cell-mediated immunity D. immediate hypersensitivity

Correct Answer: A

A concentrate of lymphocytes can be prepared form peripheral blood by: A. density gradient centrifugation B. ultracentrifugation C. zone electrophoresis D. freeze fractionation

Correct Answer: A

A consistently and repeatedly negative IFA-ANA is: A. strong evidence against untreated SLE B. associated with active SLE C. characteristic of SLE with renal involvement D. associated with lupus inhibitor

Correct Answer: A

A maternal serum rubella titer that is equal to or greater than 1:8 indicates: A. probable immunity to rubella B. evidence of acute rubella infection C. susceptibility to rubella infection D. absence of acute rubella

Correct Answer: A

A newborn is to be tested for a vertically transmitted HIV infection. Which of the following test is most useful? A. HIV PCR B. CD4 count C. Rapid HIV antibody test D. HIV IgM antibody test

Correct Answer: A

A patient came to his physician complaining of a rash, severe headaches, stiff neck, and sleep problems. Laboratory test of significance were an elevated sedimentation rate (ESR) and slightly increased liver enzymes. Further questioning of the patient revealed that he had returned form a hunting trip in upstate New York 4 weeks ago. His physician ordered a serological test for lYme disease, and the assay was negative. what is the most likely explanation of these results? A. The antibody response is not sufficient to be detected at this stage B. The clinical symptoms and laboratory results are not characteristic of Lyme disease C. The patient likely has an early infection with hepatitis B virus D. Laboratory error has caused a false-negative result

Correct Answer: A

A patient deficient in the C3 complement component would be expected to mount a normal: A. Type I and IV hypersensitivity B. Type II and IV hypersensitivity response C. Type I and III hypersensitivity response D. Type II and III hypersensitivity response

Correct Answer: A

A patient receives a transfusion of packed red cells and fresh frozen plasma and develop an anaphylactic, nonhemolytic reaction. She reports receiving a transfusion 20 years earlier. She had no reaction to the previous transfusion, but she did feel "poorly" a few weeks later. Which of the following transfused substances most likely elicited the reaction? A. IgA B. Group A antigen C. Rho (D) antigen D. An antigen belonging to the Duffy system

Correct Answer: A

A patient was suspected of having a lymphoproliferative disorder. After several laboratory test were completed, the patient was found to have an IgMk paraprotein. In what sequence should the laboratory test leading to this diagnosis have been performed? A. Serum protein electrophoresis (SPE) followed by immunofixation electrophoresis (IFE) B. Immunoglobulin levels followed by SPE C. Total lymphocyte count followed by immunoglobulin levels D. Immunoglobulin levels followed by urine protein electrophoresis

Correct Answer: A

78. A serum sample is positive for HBsAg. This result indicates that the person from whom the serum was taken A. Had a hepatitis B infection in the past but overcame the infection B. Has either active or chromic hepatitis B infection C. Was immunized recently against the hepatitis B infection D. Is not infectious for the hepatitis B virus

Correct Answer: B Hepatitis B surface antigen (HBsAg) is a marker for active or chronic infection by the hepatitis B virus; it indicates ongoing viral replication. A person positive for this marker is infectious. If the person had overcome a past infection, he or she would have antibody to the surface antigen (anti-HBs) but not the surface antigen. Immunization causes formation of anti-HBs antibody, and the surface antigen would not be present in serum.

89. In patients with human immunodeficiency virus infection, immune status can be monitored by measuring the ration of: A. CD3+ cells to CD8+ B. CD4+ cells to CD8+ cells C. Lymphocytes to Monocytes D. T cells to B cells

Correct Answer: B Human immunodeficiency virus preferentially infects T helper cells, which are positive for the surface marker CD4. As the infection progresses, the number of CD4+ cells in the peripheral bloodstream decreases. CDS is a marker found on another subset of T cells, cytotoxic T cells. The reference ratio of CD4:CD8 cells is 2:1. A decrease in the ratio indicates a decline in immune function.

75. Incompatible blood transfusions are examples of A. Type 1 hypersensitivity reactions B. Type II hypersensitivity reactions C. Type III hypersensitivity reactions D. Type IV hypersensitivity reactions

Correct Answer: B Incompatible blood transfusions are examples of a type II hypersensitivity reaction. These reactions are characterized as the antigen being a part of a cell. Antibody binds to the antigen, complement is activated, and the red blood cells are lysed.

33. Which cell is the principal source of interleukin 2? A. B cell B. T cell C. Monocyte D. Plasma cell

Correct Answer: B Interleukin2(IL-2)isalymphokineproduced by activated T helper cells. IL-2 principally affects T cells, including the cell that released IL-2, acting on its target cells via the IL-2 receptor. This receptor is not present on resting cells.

56. Which of the following statements about immunoglobulin light chains is true? A. Each immunoglobulin monomer has either one kappa or one lambda chain B. There are two types: kappa and lambda C. They consist of constant regions only D. They form part of the Fc fragment

Correct Answer: B Light chains are of two distinct types: kappa and lambda. Either type may combine with any of the heavy chains, but in any one molecule, only one type is found. Each immunoglobulin monomer contains two light chains, either kappa or lambda. They extend into the Fab, or antigen- binding, site. This half of the chain is highly variable, whereas the carboxy-terminal portion of the molecule is a constant region.

4. A lymphokine is A. A soluble mediator produced by granulocytes and affecting lymphocytes B. A soluble mediator produced by lymphocytes C. A soluble mediator produced by plasma cells D. An antibody that reacts with lymphocytes

Correct Answer: B Lymphokines are soluble mediators of immune reactions. They are produced most often by T lymphocytes. Antibodies are produced by plasma cells.

24. Multiple myeloma is a A. Lymphoproliferative disease of T cells B. Cancer of plasma cells characterized by increased antibody concentration C. Lymphoproliferative disease resulting in a decrease in antibody production D. Cancer of monocytes characterized by increased kappa and lambda chain synthesis

Correct Answer: B Plasma cells are normally end-stage cells; they live a few days and die. During multiple myeloma, plasma cells become cancerous and continue to secrete antibody. The cells also secrete excess light chains that can be found in the urine; these proteins are called Bence Jones proteins.

42. The agglutination pattern shown in Color Plate 23 was observed while performing an antibody titration. This agglutination pattern is an example of A. A prezone reaction B. A prozone reaction C. A postzone reaction D. Incomplete complement inactivation

Correct Answer: B Prozone occurs when an extremely high titer of antibody is present. In the first tubes of the titration, not enough antigen is present to allow for cross-linking and lattice formation. The antibody effectively blocks all the antigen sites present, so agglutination does not occur. Complement is not involved in antibody titration.

60. Skin testing is a useful diagnostic tool in a number of disorders, such as tuberculosis. Which of the following statements about skin testing is true? A. A positive test depends on preformed antibody B. Reactivity to a particular antigen may be transferred from one individual to another by sensitized lymphocytes C. The intensity of the response correlates directly with the clinical activity of the disease D. The maximum response will occur immediately

Correct Answer: B Skin testing is based upon the presence of T cells sensitized to antigen. Their activation produces a delayed hypersensitivity reaction, which reaches its peak in about 48 hours. There is no correlation of the amount of the reaction with clinical disease. If the sensitized T cells are transferred from one individual to another, the recipient individual will manifest the same delayed hypersensitivity as the donor.

85. "Superantigens" are toxins produced by some stains of Staphylococcus aureus and group A streptococci and cause damage by A. Molecular mimicry B. Polyclonal T cell activation C. Lysin white blood cells and platelets D. Lysing red blood cells

Correct Answer: B Some strains of Staphylococcus aureus and group A streptococci produce toxins that have the properties of "superantigens." Superantigens react with T cells directly without processing by an antigen presenting cell. These toxins can stimulate many T cells, rather than only those T cells bearing T cell receptors specific for the bacterial toxins. The result is a massive T cell response, leading to the release of cytokines and resulting in disease entities known as toxic shock syndrome (in the case of S. aureus infection) and toxic shock-like syndrome in the case of group A streptococci.

63. Which of the following is a characteristic of T cells? A. Synthesize antibody B. Mature in the thymus C. Able to bind unprocessed antigen D. Primarily protect against extracellular parasites

Correct Answer: B T cells are produced in the bone marrow and mature in the thymus. Plasma cells, not T cells, produce antibody, and T cells can only react to antigen processed by an antigen-presenting cell. The cell-mediated immune response, which requires the activity of T cells, is primarily helpful in fighting against intracellular parasites.

97. An 11 year-old female presents with fever, sore throat, lethargy, and tender cervical lymphademopathy. Relevant findings include splenomegaly and lymphocytosis, with many large reactive (atypical) lymphocytes. A heterophile antibody test was negative. Further laboratory results were as follows: IgG titer; IgM titer -Cytomegalovirus: 20;0 -Epstein-Barr virus (EBV) VCA: 0;80 -Mono spot: 0;0 What conclusion can be made concerning the diagnosis? A. Acute CMV infection B. Acute EBV infection C. Chronic CMV infection D. Chronic EBV infection

Correct Answer: B The symptoms of fever of unknown origin, lymphocytosis, and lymphadenopathy suggest EBV or CMV infection and lymphoma or leukemia. Heterophile antibodies become positive later than antibodies to viral core antigen (VGA) of EBV. In addition, only about 50% of children less than 12 years of age form heterophile antibodies following EBV infection. The IgM titer of 80 for EBV is consistent with acute EBV infection.

99. A 25-year-old male presents to his family physician complaining of fatigue, diarrhea, and weight loss of a few months duration. On physical examination the patient is found to have a fever and abdominal discomfort. Laboratory results indicate a white blood cell count of 14.3 x 10^9/L (reference range 4.8-10.8 x 10^9/L). Assays for HBsAg and anti-HIV are negative. An ELISA test for antibodies to the human immunodeficiency virus (HIV) performed on the patient's serum is found to be reactive. What step should be take next? A. Call the physician with the HIV result B. Repeat the HIV ELISA test on the sample C. Test the patient's serum for anti-HBs D. Contact the patient to collect a second sample

Correct Answer: B When a standard screening test for human immunodeficiency virus infection, such as an ELISA, is positive, it is recommended that the sample be repeated in duplicate. If one or both of the repeated tests is reactive, the sample is considered to be repeatedly reactive and needs to be confirmed by a confirmatory test (e.g., Western blot or immunofluorescent antibody). If this test is positive, the sample can be reported as positive. If the confirmatory test is negative, an additional confirmatory test should be performed if the patient has risk factors for HIV infection. If the repeated ELISA tests are both negative, the sample is reported as negative.

68. An ouchterlony gel diffusion plate is depicted in Color Plate 24. The center well contains antibody, and the peripheral wells contain antigens labeled 1 through 4. What is the relationship between the antigens in wells 2 and 4? A. Cannot be determined B. They are identical C. They are unrelated D. They react incompletely with the antibody

Correct Answer: B When two antigens are identical, a smooth curved line of precipitation is formed between them. In the diagram, the antigen in well 2 is identical to one of two antigens in well 1. The same antigen in well 1 is identical to antigen in well 4. Therefore, it follows that antigens 2 and 4 are identical.

12. Which immunoglobulin(s) help(s) initiate the classic complement pathway? A. IgA and IgD B. IgM only C. IgG and IgM D. IgG only 3.1 - Basic Principles of Immunology

Correct Answer: C

17. Which immunoglobulin class(es) has (have) a J chain? A. IgM B. IgE and IgD C. IgM and sIgA D. IgG3 and IgA 3.1 - Basic Principles of Immunology

Correct Answer: C

22. All of the following are functions of immunoglobulins except: A. Neutralizing toxic substances B. Facilitating phagocytosis through opsonization C. Interacting with Tc cells to lyse viruses D. Combining with complement to destroy cellular antigens 3.1 - Basic Principles of Immunology

Correct Answer: C

25. Which of the following are products of HLA class III genes? A. T-cell immune receptors B. HLA-D antigens on immune cells C. Complement proteins C2, C4, and Factor B D. Immunoglobulin Vl region 3.1 - Basic Principles of Immunology

Correct Answer: C

31. T regulator cells, responsible for controlling autoimmune antibody production, express which of the following phenotypes? A. CD3, CD4,CD8 B. CD3, CD8, CD25 C. CD3, CD4, CD25 D. CD8, CD25, CD56 3.1 - Basic Principles of Immunology

Correct Answer: C

7. Which T cell expresses the CD8 marker and acts specifically to kill tumors or virally infected cells? A. Helper T B. T suppressor C. T cytotoxic D. T inducer/suppressor 3.1 - Basic Principles of Immunology

Correct Answer: C

9. What is the name of the process by which phagocytic cells are attracted to a substance such as a bacterial peptide? A. Diapedesis B. Degranulation C. Chemotaxis D. Phagotaxis 3.1 - Basic Principles of Immunology

Correct Answer: C

A 28-year-old man is seen by a physician because of several months of intermittent low back pain. The patient's symptoms are suggestive of ankylosing spondylitis. Which of the following laboratory studies would support this diagnosis? A. a decreased synovial fluid CH50 level B. low serum CH50 level C. positive HLA-B27 antigen test D. rheumatoid factor in the synovial fluid

Correct Answer: C

A bacterial protein used to bind human immunoglobulins is: A. HAV antibody, IgA B. Escherichia coli protein C C. staphylococcal protein A D. HAV antibody, IgG type

Correct Answer: C

A carbohydrate antigen 125 assay (CA-125_ was performed on a woman with ovarian cancer. After treatment, the levels fell significantly. An examination performed later revealed the recurrence of the tumor, but the CA 125 levels remained low. How can this finding be explained? A. Test error B. CA-125 was the wrong laboratory test; a-fetoprotein (AFP) is a better test to monitor ovarian cancer C. CA-125 may not be sensitive enough when used alone to monitor tumor development D. CA-125 is not specific enough to detect only on type of tumor

Correct Answer: C

A child has severe hay fever. A total IgE measurement was performed by the Ouchterlony immunodiffusion method. No lines of precipitation appeared on the immunodiffusion plate. The most likely explanation is: A. IgE antibodies are not produced in children who have hay fever B. hay fever is mediated by the cellular system C. IgE is in too low a concentration to be detected by this method D. IgA is the antibody commonly produced in people with hay fever

Correct Answer: C

A dialysis patient is positive for both hepatitis B surface antigen and hepatitis B surface antibody. The physician suspects a laboratory error. Do you agree? A. Yes; the patient should not test positive for both HBsAg and HBsAb B. No: imcomplete dialysis of a patient in the core window phase of hepatitis B infection will yield this result C. No: it is likely the patient has recently received a hepatitis B booster vaccination and could have these results D. Perhaps: a new specimen should be submitted to clear up the confusion

Correct Answer: C

96. A 28-year-old female complains to her family physician of abdominal pain, loss of appetite, and low-grade fever. Physical examination reveals abdominal tenderness and a low-grade fever. Her physician orders a hepatitis profile and obtains the results below. -Anti-HAV: Nonreactive -Anti-HBc: Reactive -Anti-HBs: Nonreactive -HBsAg: Reactive -HBeAg: Reactive -Anti-HCV: Nonreactive Which of the following is the most likely conclusions? A. Acute HAV infection B. Acute HBV infection C. Chronic HBV infection D. Immunity to HBV due to past infection

Correct Answer: C The presence of HBsAg indicates viral replication and that the patient is infectious; this marker can be seen in both acute and chronic infections. The lack of anti-HBs indicates that the patient is not immune to the infection. The presence of anti-HBc and HBeAg with HBsAg indicates a chronic infection.

44. The rapid plasma reagin assay for syphilis does not need to be read microscopically because the antigen is; A. Cardiolipin B. Complexed with latex C. Complexed with charcoal D. Inactivated bacterial cells

Correct Answer: C The rapid plasma reagin (RPR) and Venereal Disease Research Laboratory (VDRL) tests use a cardiolipin antigen. However, in the RPR test, charcoal particles are included with the antigen. When antibody in the patient sample combines with the antigen, the charcoal is trapped in the immune complex, allowing the reaction to be read macroscopically.

37. How does the secondary humoral immune response differ form the primary response? A. The lag phase (the time between exposure to immunogen and production of antibody) is longer in the secondary immune response. B. IgM is the predominant antibody class produced in the secondary immune response. C. The antibody levels produced are higher in the secondary immune response. D. Cytotoxic T lymphocytes play an important role in the secondary response.

Correct Answer: C The secondary immune response is characterized by the predominance of IgG over IgM. In addition, because of the formation of memory cells following the primary response, the secondary response occurs much more quickly and strongly. This is the basis for immunization as a protection against various infectious diseases. Cytotoxic T lymphocytes are not involved in humoral immunity.

94. An antibody titration is depicted in Color Plate 25. In this titration, a 0.2 mL aliquot of a patient's serum sample was added to 0.8 mL of saline, and this mixture was place into tube #1. A 0.5 mL sample was removed form tube #1 and placed into tube #2, containing 0.5 mL of saline. This procedure was repeated through tube #10. The dilutions were assayed for antibody to an infectious agent. How should the antibody titer be reported? A. 256 B. 512 C. 640 D. 1280

Correct Answer: C The titer of this assay is the reciprocal of the highest dilution demonstrating the desired result, in this case tube #8. The dilution is deter- mined as shown below. Dilution for tube #1: 0.2 mL serum in a total volume of 1.0mL =1:5 dilution. Dilutions in succeeding tubes: 0.5 mL diluted serum in a total volume of 1.0 mL = 1:2 dilution. The dilutions in the series of tubes are as follows: Tube #1, 1:5; tube #2, 1:10; tube #3, 1:20; tube #4,1:40; tube #5,1:80; tube #6,1:160; tube #7, 1:320; tube #8, 1:640; tube #9, 1:1280; tube#10, 1:2560 The reciprocal of the dilution in tube #8 (1:640) is 640.

1. From the following, identify a specific component of the adaptive immune system that is formed in response to antigenic stimulation: A. Lysozyme B. Complement C. Commensal organism D. Immunoglobulin 3.1 - Basic Principles of Immunology

Correct Answer: D

11. Which complement component is found in both the classic and alternative pathways? A. C1 B. C4 C. Factor D D. C3 3.1 - Basic Principles of Immunology

Correct Answer: D

21. Which immunoglobulin cross links mast cells to release histamine? A. IgG B. IgM C. IgA D. IgE 3.1 - Basic Principles of Immunology

Correct Answer: D

23. Which of the following cell surface molecules is classified as an MHC class II antigen? A. HLA-A B. HLA-B C. HLA-C D. HLA-DR 3.1 - Basic Principles of Immunology

Correct Answer: D

26. What molecule on the surface of most T cells recognizes antigen? A. IgT, a four-chain molecule that includes the tau heavy chain B. MHC protein, a two-chain molecule encoded by the HLA region C. CD3, consisting of sic different chains D. TcR, consisting of two chains, alpha and β 3.1 - Basic Principles of Immunology

Correct Answer: D

30. A superantigen, such as toxic shock syndrome toxin-1 (TSST-1), bypasses the normal antigen processing stage by binding to and cross linking: A. A portion of an immunoglobulin molecule and complement component C1 B. Toll-like receptor and an MHC class I molecule C. A portion of an immunoglobulin and a portion of a T-cell receptor D. A portion of a T-cell receptor and a MHC class II molecule 3.1 - Basic Principles of Immunology

Correct Answer: D

6. Which markers are found on mature, peripheral helper T cells? A. CD1, CD2, CD4 B. CD2, CD3, CD8 C. CD1, CD3, CD4 D. CD2, CD3, CD4 3.1 - Basic Principles of Immunology

Correct Answer: D

A 12-year old girl has symptoms of fatigue and a localized lymphadenopathy. Laboratory test reveal a peripheral blood lymphocytosis, a positive RPR, and a positive spot test for IM. What test should be performed next? A. HIV test by ELISA B. VDRL C. Epstein-Barr virus (EBV) specific antigen test D. Treponemal pallidum particle agglutination (TP-PA) test

Correct Answer: D

A 25-year-old woman is seen by a physician because of Raynaud phenomenon, myalgias, arthralgias, and difficulty in swallowing. There is not evidence of renal disease. An ANA titer is 1:5120 with a speckled pattern with mitotic. Which of the following are also likely to be found in this patient? A. high-level nDNA and a low CH50 level B. high-level Sm antibody C. high-titer rheumatoid factor D. high-level ribonucleoprotein (RNP) antibody

Correct Answer: D

A VDRL serum sample is heat inactivated, then placed in a refrigerator for overnight storage. Before being tested, the serum must be: A. kept colder than 10 C B. allowed to equilibrate to room temperature C. warmed to 37 C D. reheated to 56C for 10 minutes

Correct Answer: D

A cold agglutinin titer end point is 1:16 after incubating overnight in the refrigerator and remains 1:16 after warming. The best course of action is to: A. report the titer as negative B. report the titer as positive, 1:16 C. repeat the titer with a fresh sample D. test for antibody specificity

Correct Answer: D

A haptenic determinant will react with: A. both T cells and antibody B. T cells but not antibody C. neither T cells nor antibody D. antibody but not T cells

Correct Answer: D

On January 4, a serum protein electrophoresis on a specimen obtained at your hospital in North Dakota form a 58-year-old patient shows a band at the B-y junction. The specimen was also positive for rheumatoid factor. You recommend that an immunofixation test be performed to determine if the band represents a monoclonal immunoglobin. Another specimen is obtained 2 weeks later by the physician in his office 30 miles away, and the whole blood is submitted to you for the IFE. The courier placed the whole blood specimen in an ice chest for transport. In this specimen, no B-y band is seen in the serum protein lane, and the IgM lave is very faint. The rheumatoid factor on this specimen was negative. The physician wants to know what's wrong with your laboratory Your response is: A. Nothing's wrong with our laboratory; the patient had an infection 2 weeks ago that has cleared up B. Something's wrong with our laboratory-we likely mislabeled one of the specimens; please resubmit a new specimen and we will test it at no charge C. You will run a second specimen using a 2-mercaptoethanol treatment that will eliminate IgM aggregates and allow for more sensitive monoclonal IgM detection D. The physician should redraw another specimen from the patient and this time separate the serum form the clot in his office before sending the specimen in by courier

Correct Answer: D

Polyclonal B cell activation: A. inhibits antibody production B. requires the participation of T helper cells C. results form the activation of suppressor T cells D. can induce autoantibody production

Correct Answer: D

Rapid assays for influenza that utilize specimens obtained from nasopharyngeal swabs detect: A. IgM anti-influenza B. IgA anti-influenza C. IgA-influenza Ag immune complexes D. Influenza antigen

Correct Answer: D

Refer to the following illustration See BOC pg 239 Pic 2 A nonspecific precipitin reaction is demonstrated in: A. Figure #1 B. Figure #2 C. Figure #3 D. Figure #4

Correct Answer: D

Refer to the following illustration of the hepatitis B virus: See BOC pg 246 Pic 2 Select the corresponding lettered component indicated on the diagram for e antigen. A. A B. B C. C D. D

Correct Answer: D

Refer to the following results for peripheral blood samples: %T lymphocytes Patient #1: 85% Patient #2: 23% Patient #3: 51% Patient #4: 82% normal control: 44% The data above indicates: A. patient #1 has an abnormally high T lymphocyte count B. patient #2 has a normal T lymphocyte count C. patients #1 and #3 have normal T lymphocytes D. the normal control is too low and another sample should be selected

Correct Answer: D

Rheumatoid factor reacts with: A. inert substances such as latex B. Rh-positive erythrocytes C. kinetoplasts of Crithidia luciliae D. gamma globulin-coated particles

Correct Answer: D

The IgM molecule is a: A. dimer B. trimer C. tetramer D. pentamer

Correct Answer: D

The absorbance of a sample measured by ELISA is greater than the highest standard. What corrective action should be taken? A. Extrapolate an estimated value from the highest reading B. Repeat the test using a standard of higher concentration C. Repeat the assay using one half the volume of the sample D. Dilute the test sample

Correct Answer: D

The assembly of the complement "membrane attack unit" is initiated with the binding of: A. C1 B. C3 C. C4 D. C5

Correct Answer: D

The best method to detect infections due to rubella, Epstein-Barr and human immunodeficiency viruses is: A. antigen detection by EIA B. cell culture C. antigen detection by Western blot D. antibody detection by EIA

Correct Answer: D

The curve below was obtained by adding increasing amounts of a soluble antigen to fixed volumes of monospecific antiserum: See BOC pg 237 Pic 2 The area on the curve where soluble antigen-antibody complexes have begun to form is: A. A B. B C. C D. D

Correct Answer: D

The detection of precipitation reactions depends on the presence of optimal proportions of antigen and antibody. A patient's sample contains a large amount of antibody, but the reaction in a test system containing antigen is negative. What has happened? A. Performance error B. Low specificity C. A shift in the zone of equivalence D. Prozone phenomenon

Correct Answer: D

The following procedure has been routinely used for detection of hepatitis B surface antigen (HBsAg) because of its high level of sensitivity: A. hemagglutination B. counterimmunoelectorphoresis C. radial immunodiffusion D. ELISA

Correct Answer: D

The immunoglobulin class associated with immediate hypersensitivity or atopic reactions is: A. IgA B. IgM C. IgD D. IgE

Correct Answer: D

What antibodies are represented by the nucleolar pattern in the immunofluorescence test for antinuclear antibodies? A. Antihistone antibodies B. Anti-dsDNA antibodies C. Anti-ENA (anti-Sm and anti-RNP) antibodies D. Anti-RNA antibodies

Correct Answer: D

What corrective action should be taken when an indeterminate pattern occurs in an indirect IFA? A. Repeat the test using a larger volume of sample B. Call the physician C. Have another medical laboratory scientist read the slide D. Dilute the sample and retest

Correct Answer: D

What is the "M" component in monoclonal gammmapathies? A. IgM produced in excess B. u Heavy chain produced in excess C. Malignant proliferation of B cells D. Monoclonal antibody or cell line

Correct Answer: D

What is the main use of laboratory test to detect antibodies to islet cells and insulin in cases of insulin-dependent diabetes mellitus (IDDM)? A. To regulate levels of injected insulin B. To diagnose IDDM C. To rule out the presence of other autoimmune diseases D. To screen susceptible individuals prior to destruction of B cells

Correct Answer: D

What is the most likely interpretation of the following syphili serological results? -RPR: reactive -VDRL: reactive -MHA-Tp: nonreactive A. Neurosyphilis B. Secondary syphilis C. Syphilis that has been successfully treated D. Biological false positive

Correct Answer: D

What method may be used for tissue typing instead of serological HLA typing? A. PCR B. Southern blotting C. RFLP D. All of these options

Correct Answer: D

Which CD4:CD8 ratio is most likely in a patient with acquired immunodeficiency syndrome (AIDS)? A. 2:1 B. 3:1 C. 2:3 D. 1:2

Correct Answer: D


Ensembles d'études connexes

Chapter 40: Management of Patients with Gastric and Duodenal Disorders

View Set

NUR 2449 Mastery Level 3: Basic Care and Comfort

View Set

Comp Exam 2 study Ch 5, 6, 7, 8, 17

View Set

Individualism, Modern Capitalism, and Dystopian Visions

View Set

Phrases pour communiquer en français

View Set